Вы находитесь на странице: 1из 170

Financial Accounting

Author Dr. Guruprasad Murthy

Himalaya Publishing House (Private) Limited Mumbai

Student Manual

FINANCIAL ACCOUNTING BY DR. GURUPRASAD MURTHY STRUCTURE OF PROBLEMS AND SOLUTIONS (CD ROM)

Module Nos. Module 1

Assignment Nos.
Assignment 1 38

Title / Topics
Problem and Solutions Equation Method

CD ROM Pages
03 52

List of Assignments Assignment 3.1 Assignment 3.2 Assignment 3.3 Assignment 3.4 Assignment 3.5 Assignment 3.6 Assignment 3.7 Assignment3.8

Rashid Enterprises Thomas Sons M/s. X Company Ltd. M/s. SPJ & Co. Ltd. M/s. Dalvi Co. Ltd. M/s. Ashutosh Abacus Airways Ltd. M/s. PQR

53 - 63 64 - 69 70 89 90 - 110 111 - 130 131 141 142 159 160 - 169

Module: 1 - Problems and Solutions

Assignment 1
Match the following expressions in column 1 and 2 Column 1 Balance sheet Assets Liabilities Capital Column 2 Owners Funds Financial Statement Uses of funds Borrowed funds

Answer to Assignment 1
Column 1 Balance sheet Assets Liabilities Capital Column 2 Financial Statement Uses of funds Borrowed funds Owners' Funds

Assignment 2
(pertains to transactions 1, 2 and 3) Assets are _____________________ of funds; liabilities are __________________ of funds; capital is ______________________ of funds.

Answer to Assignment 2 (pertains to transactions 1, 2 and 3)


Assets are uses of funds; liabilities are sources of funds; capital is source of funds.

Assignment 3 (pertains to transactions 4 and 5)


Identify the attributes of Fixed Assets Fixed assets are: a)_______________________________________________________________________________ b)______________________________________________________________________________ c)______________________________________________________________________________ d)______________________________________________________________________________ e)______________________________________________________________________________ f)______________________________________________________________________________

Answer to Assignment 3 (pertains to transactions 4 and 5)


Identify the attributes of Fixed Assets Fixed assets are: a) Long term uses of funds or long term assets owned by a business

b) Expenditure on capital account that is to say they are capital expenditures c) Assets, which are acquired to derive benefits over a long period of time
d) Made up of tangible assets viz. Land, buildings, plant and machinery, furniture and fixtures and vehicles and intangible assets viz. goodwill, patents, copyrights and trademarks

d) Made up of movable and immovable items


f) Assets of a permanent, durable and lasting nature and may be acquired through cash or on credit, depending on the specific transaction entered into.

Assignment 4 (pertains to transactions 1 to 5)


From the five transactions so far, what general observations can we make regarding any financial statements and what concepts emerge from the observations? -_________________________________________________________________________ -_________________________________________________________________________

Answer to Assignment 4 (pertains to transactions 1 to 5)


From the five transactions so far, what general observations can we make regarding any financial statements and what concepts emerge from the observations? Observation Concept Every item is measured in terms of money and money only Every transaction has a two fold effect: 1.Once on the asset side and once on the liability side (transaction 1,2,3 and 5) 2.Increase and decrease on asset side (Transaction 4) Owners and business are separate entities. Money measurement Dual aspect

Entity concept

Assignment 5 (pertains to transaction 6) 5.1. Stocks are sold with a view to make a ________________ and convert into ______________ as soon as possible. Two items have emerged so far under current assets, viz, stock or inventories (finished goods) and cash. 5.2. In a trading business, stocks or inventories include finished goods. In a manufacturing business inventories include:

Answer to Assignment 5 (pertains to transaction 6)


5.1 Stocks are sold with a view to make a Profit and convert into Cash as soon possible. 5.2 In a trading business stocks include finished goods, in a manufacturing business inventories include: Raw Materials Packing Materials Finished Goods

Work- in - Process Consumables: Jigs, tools & fixtures; lubricants, oil & cotton waste

Assignment 6
Given transactions 6 and 7, review your know-how on assets by filling the blanks below: Fixed Assets a) uses of fund a) Current Assets uses of funds

b)

expenditure

b)

expenditure

c) Produce

-term benefits

c)

Produce

- term benefits

d) Include

d)

Include

e)

e)

f)

f)

Answer to Assignment 6
Given transactions 6 and 7, review your know-how on assets by filling the blanks below: Fixed Assets a. Long term uses of funds b. Capital expenditure c. Produce long term benefits d. Include: 1. Land & Building 2. Plant & Machinery 3. Furniture & Fixtures 4. Patents & Copyrights 5. Trademarks e. Tangible & Intangible items; Movable & Immovable items; Animate & Inanimate items. f. 1) May be acquired on credit or against cash 2) May be owned or leased f. 1.Gross working capital = current assets 2.Net working capital = current assets minus current liabilities Current Assets a. Short term uses of funds, which should be converted into cash soon. b. Revenue expenditures c. Produce short term benefits d. Include: 1.Cash 2. Sundry Debtors or Accounts Receivables 3. Stock or Inventories

e. Represents the inputs required for the day to day running of business and are known as working capital

Assignment 7 (based on transactions 9 and 10)


7.1 Transaction 9 is a ______________________ sale unlike transaction 10, which is a _________________ sale. The principle of recognizing sales as an income remains the same. However, the detailed implications are driven by the nature of the transaction.

7.2 Prepare an Income statement for both the sales transactions (9 and 10) taken together.
Sales

Cash sales Credit sales Total sales Less: Cost of Goods sold Gross Trading Profit

Answer to Assignment 7 (based on transactions 9 and 10) 7.Transaction 9 is a credit sale unlike transaction 10, which is a cash sale.
The principle of recognizing sales as an income remains the same. However, the detailed implications are driven by the nature of the transaction. 7.1 Prepare an Income statement for both the sales transactions (9 and 10) taken together. Income statement for the period ending 10.01.2000 Sales Cash sales 4000 bottles @ Rs. 20 per bottle Credit sales 6000 bottle @ Rs. 20 per bottle Total Sales Less: Cost of Goods sold or cost of sales (10000 bottles @ Rs. 10 per bottle) Gross Trading Profit Rs. 80,000 1,20,000 Rs. 2,00,000 (1,00,000) 1,00,000

Assignment 8 (Conceptual question)


8.1 We have purchased 14,000 bottles in all so far: Transaction 6 Transaction 7 Transaction 8 Why? Please explain? Purchases Purchases Purchase returns 10,000 bottles 5,000 bottles (1,000) bottles

However we have charged the income statement with only 10,000 bottles?

_____________________________________________________________________________________________________________________________________

10

8.2 Differences between transactions 9 and 10.

Transaction 9 1) 2) 3)

Transaction 10

8.3. Profits = Liability. Provide conceptual justification.

8.4. If Profits = Liability, then Loss = Assets. Again, provide conceptual justification for Loss = Asset

8.5. Distinguish between Profit and Loss Profit 1 2 3 4 5 1) 3) 5) 7) 9) 2) 4) 6) 8) 10) Loss

8.6 State the attributes of Assets and Liabilities Assets 11) 13) 12) 14) Liabilities

11

15) 17) 19)

16) 18) 20)

Answer to Assignment 8 (Conceptual question)


8.1 Purchased 14,000 bottles, charged only 10,000 bottles to the Income Statement. The profits are computed applying the 'matching concept', which matches relevant revenues with relevant expenses. Relevant revenues 10,000 bottles sold, therefore relevant expenses purchase cost of 10,000 bottles. Till such time as the remaining bottles are sold, they continue to remain as an inventory in the current assets section of the balance sheet. 8.2 Difference between transaction 9 and 10 Transaction 9 1) 2) 3) Credit sale Profits have increased by Rs. 0.6 lakhs but cash remains unaffected. This is a book profit. However, debtors have emerged and equal Rs.1.2 lakhs. Risk of bad debts i.e., Customer who will default or risk of doubtful debts i.e., Customers who may not pay. Cash sale Profits have increased by Rs. 0.4 lakhs. This is a book profit and cash profit. However, cash in the business has increased by Rs. 0.8 lakhs. No risk of bad debts

Transaction10

8.3 Profits = Liability. Provide conceptual justification. Entity concept distinguishes between the owner and the business as two distinct and separate entities. Capital contributed by owners was recorded on the liabilities side (transaction 1 and 2). It means that M/s Aryan Traders are obliged to the owner to the extent of the profit. Thus, at transactions 1 and 2, the business was obliged to the owners to the extent of Rs. 12 lakhs (preference share capital Rs. 2 lakhs and equity share capital Rs. 10 lakhs). If the business was liquidated at that point of time, the said shareholders would receive their respective contribution in that order. Now, at transaction 10, Aryan Traders have made a profit of Rs. 1 lakh. The obligations of the owners have increased by Rs. 1 lakh. Hence, Profit = Liability based on the entity concept. 8.4 If Profit = Liability, then Loss = Assets Again, provide conceptual justification for Loss = Asset Using the entity concept, profit increases the obligation of business to the owners. Loss decreases the obligation of business to the owners. In the event of liquidation, the owners can get back their capital only after adjusting for the loss amongst other obligations of business. Hence, loss is an asset. 8.5 Distinguish between Profit and Loss

12

Profit 1 2 3 4 5 Profits refer to the excess of income over expenses

Loss Losses refer to the excess of expenses over income Losses reduce the net worth of the business Losses are recorded on the asset side of the balance sheet Losses do not do so

Profit increases the net worth of the business

Profits are recorded on the liability side of the balance sheet Profits signify the well being of the company

Profit is a liability.

Loss is an asset.

8.6 Attributes of assets and liabilities

Assets 1 2 3 4 The economic resources owned by a company Properties and possession of every description. Short term and long term uses of funds. Unexpired utility utility of fixed assets are spread over a long period of time and utility of current assets expire within a short period, usually a year. Obligations of every description.

Liabilities The economic resources owed by a company Short term and long term sources of funds. Obligations, which have not matured - long-term obligations mature over a long period of time and short-term obligations (current liabilities) mature within a short period of time, usually a year. Capital contributed by the shareholders is permanent capital. Equity shareholders get back their capital only on the company winding up business. Preference shareholders get back their capital in accordance with the contract.

Assignment 9
Test your thoughts on current assets: 9.1 Current assets include ______________, ___________________ and __________________. 9.2 Current assets are _________________ uses of funds. 9.3 Current assets are those, which are to be __________________ into ___________________________________________________. 9.4 Current assets represent __________________ capital of any business and are required for ________________________ operations of the business.

13

Answer to Assignment 9
Test your thoughts on current assets: 9.1 Current assets include cash, debtors or accounts receivables and stock or inventories. 9.2 Current assets are short-term uses of funds. 9.3 Current assets are those, which are to be converted into cash as soon as possible. 9.4 Current assets represent working capital of any business and are required for day- to- day operations of the business.

Assignment 10
Prepaid Insurance is:

Answer to Assignment 10
Prepaid Insurance is: Pre-paid Insurance is a current asset. Prepaid Insurance represents the right to receive benefits from the Insurance Company during the tenure of the policy. As and when 2001 arrives, prepaid insurance will be treated as an expense.

Assignment 11
OBSERVE from Transaction 12 Cash by Rs.0.5 lakh. Expense by Rs.0.24 lakh only. Why has the expense not by the full amount of Rs.0.5 lakh. Explain with the help of an accounting principle.

Answer to Assignment 11
OBSERVE from Transaction 12 Cash by Rs. 0.5 lakh. Expenses by Rs. 0.24 lakh only. Why has the expense not by the full amount of Rs. 0.5 lakh. Explain with the help of an accounting principle.

14

The accounting principle governing prepaid insurance is the 'accrual principle' and can be expressed as follows:

'An expense is not incurred merely because cash is paid'.


Further, the matching concept which requires matching relevant expenses with the appropriate accounting period is also applicable Rs. 0.24 lakh expensed in 2000. Rs. 0.26 lakh to be expensed in 2001. Pre-paid Insurance represents a payment in advance of the commencement of the next accounting period. To reiterate, such an advance payment is recognized as prepaid insurance, which is part of current assets section of the balance sheet.

Assignment 12
In Transaction 12, Cash by Rs._________; Expense by Rs._________ and Reserve by Rs._________. Accounting Principle

Answer to Assignment 12
In Transaction 12, Cash by Rs. 0.5 lakh; Expenses by Rs. 0.24 lakh and Reserve by Rs. 0.24 lakh. Accounting principle: Accrual concept & Matching concept.

Assignment 13
From Transaction 14, cash increases. However, profits and reserves are unaffected. Explain why. (Hint: Accrual principle express in words) ______________________________________________________________________________________________ ______________________________________________________________________________________________

Answer to Assignment 13
From Transaction 14, cash increases alright. However, profits and therefore reserves are unaffected. Explain why. (Hint: Accrual principle express in words) An income is not earned merely because cash is received.

Assignment 14
Money is received today, goods/services to materialize at a future date. Explain the accounting logic, which justifies parking the cash received as income received in advance or pre-received income.

15

Answer to Assignment 14
Money is received today, goods/services to materialize at a future date. Explain the accounting logic, which justifies parking the cash received as income received in advance. Matching concept matching incomes relevant to the accounting period in question. The goods or services are to materialize to the customer at a future date. Hence, postpone or defer the incomes to a future relevant period in which the matching relevant expenses emerge.

Assignment 15
15.1 Conceptualize: Transaction 16 Money has not been paid. Expense has been recognized and booked. Profits by the amount of Rs.1 lakh. Accounting Principle:

Between transaction 1 to transaction 10, we had prepared an Income Statement to compute the profit. 15.2. Complete the Income Statement to reflect transactions for the period ending 20.01.2000 (Transactions 1 to 16). Income Statement for the period ending 20.01.2000 (Transactions 1 to 16) Sales Cash sales Credit Sales Total Sales Less: Cost of Goods Sold Gross Trading Profit Less: Non-trading expenses Rent Insurance

Answer to Assignment 15.1


Conceptualize: Money has not been paid. Expense has been recognized and booked. Profits to by the amount of Rs. 1 lakh. Accounting Concept: Accrual Concept again: 'An expense is incurred whether cash is paid now or later'. Matching principle too- match relevant revenues with relevant expenses.

16

Answer to Assignment 15.2


Between transactions 1 to transaction 10, we had prepared an Income Statement to compute the profit. 15.Complete the Income Statement to reflect transactions for the period ending 20.01. 2000 (Transactions 1 to 16). Income Statement for the period ending 20.01.2000 (Transactions 1 to 16) Tr. 10 Tr. 9 Cash sales 4,000 bottles @ Rs.20 per bottle Credit sales 6,000 bottles @ Rs.20 per bottle Total Sales Tr. 6 Less: Cost of goods sold 10,000 bottles @ Rs 10 per bottle Gross Trading Profit Less: Non-trading expense Tr. 11 Tr. 16 Tr. 12 Tr. 12 Rent : Incurred and paid Incurred but not paid Insurance paid Less: Prepaid insurance Loss Tr. 13 Add: Income other than sales Loss 0.5 1.0 0.5 (0.26) 0.24 (0.74) 0.25 (0.49) (Rs lakhs) 0.8 1.2 2.0 (1.0) 1.0

17

Assignment 16
What concepts have been followed in constructing the Income Statement upto Transaction 16? 1.With respect to sales (Transaction 9) 2. With respect to insurance (Transaction 12) 3. With respect to income received in advance (Transaction 14) 4. With respect to rent expense (Transaction 16) 5. With respect to loss (Transaction 16) 6. Any other accounting concept/s followed. ______________________________________________________________________________________________________________________________________

Answer to Assignment 16
What accounting concepts have been followed in constructing the Income Statement upto Transaction 16? 1) Sales (transaction 9): Income is earned whether cash is received now or later. (Accrual and matching concept) 2) Insurance (transaction 12): An expense is not incurred merely because cash is paid. (Accrual and matching concept) 3) Pre-received Income (transaction 14): an income is not earned merely because cash is received. (Accrual and matching concept) 4) Rent expenses (transaction 16): An expense is incurred whether cash is paid now or later. 5) Loss: Entity concept owners and business are two different and separate entities. Remember loss is an asset. (Entity concept) 6) Other Account concept/s: Money measurement and going concern.

Assignment 17
List at least three reasons why depreciation is charged.

Answer to Assignment 17
List the reasons why depreciation is charged. 1) To recognize the wear and tear of fixed assets due to use.

18

2) To compute profit defined as 'revenue expense' where expenses include amongst other things depreciation. 3) To provide resources for replacement. Assignment 18
With the help of the above implication of depreciation in transaction 18, define: 18.1. Gross Fixed Assets in words and in figures. 18.2. Net Fixed Assets in words and in figures.

Answer to Assignment 18
With the help of the above implication of depreciation in transaction 18, define: 18.1. Gross Fixed Assets in words and in figures. Gross fixed assets represent the purchase price of fixed assets. In this case, the Gross Fixed assets are Rs. 12 lakh. 18.2. Net Fixed Assets in words and in figures. Net fixed assets are gross fixed assets adjusted for accumulated depreciation. Gross Fixed assets Less: Accumulated Depreciation Net Fixed Assets Learn more about depreciation: Consider the expression current depreciation and accumulated depreciation. Current depreciation is the depreciation charged to the income statement of a particular accounting period. Accumulated depreciation is the sum of the current depreciation charged on a year on year basis. In the first year of the fixed assets life, current depreciation amount will always equal accumulated depreciation. Rs. 12 lakhs (Rs, 2.4 lakh) Rs. 9.6 lakhs

Assignment 19
In the case on hand, if the same amount of depreciation is to be charged every year, that is for say the years ending 2001, 2002 & 2003, what will be the relevant amounts for the following items against the respective dates:

19

Year Ending 31 December Gross Fixed Assets Current Depreciation Accumulated Depreciation Net Fixed Assets Gross Fixed Assets as on 1st January

st

2000 12 .0 2.4 2.4 NIL

Rs. lakhs 2001 2002

2003

Answer to Assignment 19
In the case on hand, if the same amount of depreciation is to be charged every year, that is to say the years ending 2001, 2002 & 2003, what will be the relevant amounts for the following items: Rs. Lakhs Year Ending 31st December Gross Fixed Assets Current Depreciation Accumulated Depreciation Net Fixed Assets Gross Fixed Assets as on 1st January 2000 12.0 2.4 2.4 9.6 NIL 2001 12.0 2.4 4.8 7.2 12.0 2002 12.0 2.4 7.2 4.8 12.0 2003 12.0 2.4 9.6 2.4 12.0

Assignment 20
One of the purposes of providing for depreciation is to generate resources for replacement. What is the accounting principle that emerges?

Answer to Assignment 20
Going Concern Concept: Enterprise has an indefinite life and enjoys perpetual succession. Shareholders may come and shareholders may go, but enterprise will go on forever or at least in the foreseeable future.

20

Assignment 21
Accounting Principle followed with respect to accrued commission income is:

Answer to Assignment 21
Accounting Principle followed with respect to accrued commission income is: Accrual Principle. 'an income is earned whether cash is received now or later' Matching Principle to-match relevant revenues with relevant expense.

Assignment 22
Given the following four dimensions of accrual concept, identify the transactions which fit into the relevant slots. Accrual Concept Transactions 1. 2. 3. 4.

An expense is incurred whether cash is paid now or later


An expense is not incurred merely because cash is paid An income is earned whether cash is received now or later An income is not earned merely because cash is received

Answer to Assignment 22
Given the following four dimensions of accrual concept, identify the transactions which fit into the relevant slots.

Accrual concept An expense is incurred whether cash is paid now or later Transaction 16 Transaction 12

Transactions

An expense is not incurred merely because cash is paid


An income is earned whether cash is received now or later

Transactions 9 & 19

21

Accrual concept An income is not earned merely because cash is received

Transactions Transaction 14

Assignment 23
Revise the list of Current Assets and Current Liabilities by presenting as exhaustive a list as possible Current Assets 1. 2. 3. 4. 5. 6. 7. 8. 9. 10. 11. 12. Current Liabilities

Answer to Assignment 23
Revised list of current assets and current liabilities Current assets Cash Stock (finished goods) Prepaid insurance Accrued commission Advance income tax Current liabilities Bank or other short term loans Income received in advance or pre-received income. Outstanding expenses Provision for income tax Provision for dividends.

Sundry debtors or accounts receivables Sundry creditors or accounts payable

22

Assignment 24
State whether the following statements are true or false .Give reasons. True False

24.1 24.2 24.3 24.4

When sales increase, profits need not go up.

When sales increase, cash necessarily increases.

When expenses increase, cash necessarily decreases by the same amount. When cash is paid towards an item of expense, profits necessarily decrease by the same amount.

23

Answer to Assignment 24
Say whether the following statements are true or false. Give reasons. True 24.1When sales increase, profits need not go up. May be true 24.2 When sales increase, cash necessarily increases. May be true 24.3 When expenses increase, cash necessarily decreases by the same amount. May be true 24.4 When cash is paid towards an item of expense, profits necessarily decrease by the same amount. May be true May be false May be false May be false May be false False

24

When sales increases, profits need go up - may be true, Profits = Revenue Expense, provided the revenue is > than expense. However, if revenue is < than
may be false expense, a loss is incurred. Therefore when sales increases, profits may or may not necessarily increase.

When sales increases, cash necessarily increases may


be true, may be false

If sales have been made against cash, an increase in sales is necessarily followed by a cash increase (transactions 10, 17, and 24). If sales are made on credit, an increase in sales will not be necessarily followed by a cash increase. (Transaction 9) pending the arrival of cash, the sales amount is parked as accounts receivables or sundry debtors. If cash is paid towards an expense for the current period in question, as and when an expense increases, cash necessarily decreases. If cash is to be paid at a future date for any expense incurred, during the current accounting period, increase in expense will not cause cash decrease. Pending the payment of cash, the amount of the expense incurred is parked as outstanding expenses in the current liabilities section. Accrual principle an expense is incurred whether cash is paid now or later. Matching concept expenses relevant for an accounting period has to be charged to that period regardless of the cash payment. If cash is paid towards an expense for the current period in question, as and when cash is paid, profits will decrease. However, if cash is paid for expenses of the future period along with the current period, the amount relevant for the current period has to be separated from the amount pertaining to the future period. While the profits will decrease to the extent of current period's cash expenses, the amount pertaining to future period/s will not impact profits. Accrual principle an expense is not incurred merely because cash is paid. Matching concept expense pertaining to future periods need not be charged in the current period. Such payments are known as prepayments and are recorded as prepaid expenses (current assets section).

When expenses increase, cash necessarily decreasesmay be true, may be false

When cash is paid towards an expense, profits necessarily decrease may be true, may be false

25

Assignment 25
What is the accounting principle governing transaction 20?

Answer to Assignment 25
What is the accounting principle governing transaction 20? Accrual principle an expense is incurred whether cash is paid now or later. Matching principle to match relevant revenues with relevant expenses.

Assignment 26
State whether the following items are assets or liabilities. Item Outstanding expenses Prepaid expenses Accrued income Assets Liabilities

Pre-received income

Answer to Assignment 26
State whether the following items are assets or liabilities. Item Outstanding expenses Prepaid expenses Accrued income Assets/Liabilities Current liabilities Current assets Current assets Current liabilities

Pre-received income

Assignment 27

26

The process of parking a revenue item like advertising as a miscellaneous asset is known as capitalising. Other revenue items, which may be capitalised, include: _________________________________; _________________________________; _________________________________ Advertising expenditures is incurred in huge amounts running into millions of rupees. The benefits of these expenditures are spread over more than one accounting period. Hence, they are capitalized and spread over more than one accounting period.

Answer to Assignment 27
The process of parking a revenue item like advertising as a miscellaneous asset is known as 'capitalizing'. Other revenue items, which may be capitalized, include: Training and Development Repairs Preliminary Expenses. Advertising and the above mentioned expenditures incurred in huge amounts running into millions of rupees. The benefits of these expenditures are spread over more than one accounting period.

Assignment 28
Identify the accounting concept governing the spread of advertising expenditure over two accounting periods.

Answer to Assignment 28
Identify the account concept governing the spread of advertising expenditure over two accounting periods. Matching concept The expenditure brings benefits spread over two accounting periods. Matching relevant incomes with relevant expenses is necessary to present a true and fair picture of financial position of business.

Assignment 29 Distinguish between capital expenditures and revenue expenditures.


Particulars Capital expenditure Revenue expenditure

27

Answer to Assignment 29
Distinguish between Capital Expenditure and Revenue Expenditures Comparability: Both involve an outlay of funds, which results in either an outflow of cash or the creation of obligation with third parties as a result of a credit transaction. Distinction: Capital Expenditure Revenue Expenditure a) b) c) d) Result in the acquisition of fixed assets of a permanent, durable Result in the acquisition of current assets of a non-permanent and lasting nature. nature Non-recurring, non-routine and non-reversible. Recurring, routine and reversible. Benefit are spread over a period of time i.e., they extend into Benefits usually expire within a year. futurity. Not fully chargeable, to the profit and loss account but Fully chargeable, to the profit and loss account, unless decided expenditure is spread over the benefit producing periods of the otherwise which represents an exception to the general rule. future. Expenditure into cash through sales of fixed assets. Objective is to use the fixed assets over a long period of time. Capital assets appear on the assets side of the balance sheet. Assets into cash at the earliest opportunity. Revenue expenditures charged to profit and loss account. Only deferred portions of revenue expenditures appear on the assets side of the balance sheet usually as miscellaneous assets.

e) f)

Assignment 30
Explain the meaning of Deferred Revenue Expenditures.

Answer to Assignment 30
Explain the meaning of Deferred Revenue Expenditures Item that has been initially recorded as an asset and is expected to generate revenues over time or through the normal operations of the business is known as deferred revenue expenditure. Example: outlays on advertising, or training and development.

Assignment 31
Sometimes a customer could be a bad debt for other reasons too: 1) _________________________________________________________________________________________________________

28

2)_________________________________________________________________________________________________________

Answer to Assignment 31
Sometimes a customer could be a bad debt for other reasons too: Malafide intentions intent to deceive Indiscriminate credit policy which has not vetted the customer credentials and credit worthiness.

Assignment 32
The accounting concept guiding the charge of bad debts and provision for doubtful debts is:

Answer to Assignment 32
The accounting concept guiding the charge of bad debts and provision for doubtful debts is: Principle of conservatism or prudence. Anticipated profits ignore Anticipated losses provide for immediately. N.B. The accountants world over adopt an approach motivated by abundant caution.

Assignment 33
Identify the accounting concepts explored so far:

Answer to Assignment 33
Money measurement Entity concept Dual aspect Going concern Conservatism Accrual conceptual Matching concept Accrual conceptual Matching concept.

29

Assignment 34
Prepare an Income Statement for the period ending 31.12.2000.

(Transactions 1 to 25).
Amount Amount

Sales (transaction 9) Sales (transaction 10) Sales (transaction 17) Sales (transaction 24) Less: sales discount (transaction 24) Net Sales

30

34.1 (contd) Less: Trading cost of goods sold (schedule 1) Gross trading profit Less: Non-trading expenses (as per schedule 3) Operating loss Add: Income other than sales Commission income (transaction 13) Accrued commission (transaction 19) Advance received from customer (transaction 14) Less: Amount relevant for next year (transaction 14) Profit before income tax Less: Provision for income tax (transaction 25) Profit After income tax

31

Schedule 1 showing Trading cost of goods sold for the period ending 31.12.2000 (Rs. Lakhs) Particulars Opening Stock Add: Purchases (transaction 6) Purchases (transaction 7) Purchases (transaction 23) Less: Purchase return (transaction 8) Less: Purchase discount (transaction 23) Purchase cost of goods available for sale Less: Closing stock (as per schedule 2) Trading cost of goods sold Amount Amount

32

Schedule 2 showing closing Stock as on 31.12.2000 Rs. Lakhs

33

Schedule 3 showing non-trading expenses for the year ending 31.12.2000 Particulars Particulars Units Rent (transaction 11) Purchase Add: Outstanding rent (transaction 16) Purchase Insurance (transaction 12) Purchases Less: Prepaid (transaction 12) Purchase return Depreciation Total Plant and machinery (transaction 18) Sale Furniture and fixtures (transaction 18) Sales Interest on ICICI loan (transaction 20) Sales Bad debts (transaction 22) Sale Provision for doubtful debts @ 10% (transaction 22) Closing stock Advertising expenses (transaction 21) Less: Deferred portion (transaction 21) Total Amount Amount

(in Rs. lakhs) (in Rs. lakhs) Cost price Amount

34

Answer to Assignment 34:


Prepare an income statement for the period ending 31.12.2000 (Transactions 1 to 25) M/s Aryan Traders Limited Income and order statement for the period ending 31.12.2000 Amount Sales (transaction 9) Sales (transaction 10) Sales (transaction 17) Sales (transaction 24) Less: Sales discount (transaction 24) 1.2 0.8 1.0 2.0 (0.2) 4.8 (Rs. Lakhs) Amount

Less: Trading cost of goods sold (Schedule 1) Gross trading profit Less: non-trading expenses (as per schedule 3) Operating loss Add: Income other than sales Commission income (transaction 13) Accrued commission (transaction 19) Advance received from customer (transaction 14) Less: Amount relevant for next year (transaction 14) Profit before income tax Less: Provision for income tax (transaction 25) Profit after income tax 0.25 5.0. 1.0 (1.0)

(2.2) 2.6 (5.17) (2.57)

5.25

2.68 (0.8) 1.88

35

Schedule 1 showing trading cost of goods sold for the period ending 31.12.2000 (Rs. Lakhs) Particulars Opening stock Add: purchases (transaction 6) Purchases (transaction 7) Purchases (transaction 23) Less: Purchase returns (transaction 8) 1.0 0.5 1.0 (0.1) 2.4 Less: purchase discount (transaction 23) Purchase cost of goods available for sale Less: Closing stock (as per schedule 2) Trading cost of goods sold (0.1) 2.3 2.3 (0.1) 2.2 Amount Amount nil

36

Schedule 2 showing closing stock as on 31.12.2000 Particulars Purchase (Tr. 6) Purchase (Tr. 7) Purchase (Tr. 23) Purchase returns (Tr. 8) Total Sales (Tr. 9) Sales (Tr. 10) Sales (Tr. 17) Sales (Tr. 24) Closing stock Units 10,000 5,000 10,000 (1,000) 24,000 (6,000) (4,000) (3,000) (10,000) 1,000 10 10 10 10 10 Cost price 10 10 Amount (Rs. Lakhs) 1.0 0.5 0.9 (0.1) 2.3 (0.6) (0.4) (0.3) (1.8) 0.1

Closing Stock (in units) = Opening Stock + Purchases purchase returns Sales = Nil + [10000 + 5000 + 1000] [1000] [6000 +4000 + 3000 + 10000] = 0 + 16000 1000 14000 = 1000 UNITS

37

Schedule 3 showing non-trading expenses for the year ending 31.12.2000 Particulars Rent (transaction 11) Add: Outstanding rent (transaction 16) Insurance (transaction 12) Less: Prepaid (transaction 12) Current depreciation: Plant and machinery (transaction 18) Furniture and fixtures (transaction 18) Interest on ICICI loan (transaction 20) Bad debts (transaction 22) Provision for doubtful debts @ 10 % (transaction 22) Advertising expenses (transaction 21) Less: Deferred portion (transaction 21) TOTAL 0.5 (0.25) 0.25 5.17 2.0 0.4 2.4 0.48 0.2 0.1 Amount (Rs. Lakhs) 0.5 1.0 0.5 (0.26) 0.24 1.5 Amount (Rs. Lakhs)

38

Assignment 35
Using common sense approach, list all the cash inflows and the cash outflows. Ascertain the closing cash position of M/s Aryan Traders Ltd., as on 31.12.2000. Transaction 1 2 3 4 5 Owner brings in equity share capital Rs.10 lakhs Owner brings in preference share capital Rs. 2 lakhs M/S Aryan Traders Co. Ltd., borrowed on a long term basis from ICICI Rs. 4 lakhs @ 12 % p.a Purchased land, buildings, plant and machinery for cash Rs. 10 lakhs. Purchased furniture and fixtures Rs. 2 lakhs on credit from M/s Z 6 7 8 9 10 11 12 13 14 15 Purchased stock of 10,000 bottles of tablets @ Rs. 10 per bottle on credit (M/s P) Purchased 5,000 bottles of tablets @ Rs. 10 per bottle for cash Returns to M/s P 1,000 bottles @ Rs. 10 per bottle Sold for credit 6,000 bottles @ Rs. 20 per bottle Sold for cash 4,000 bottles @ Rs. 20 per bottle Rent expenses incurred and paid Rs. 50,000 Paid insurance for two years Rs. 50, 000 (Rs. 26,000 relates to 1990) Commission Income received in cash Rs. 25, 000 Advances received from customers Rs. 1 Lakh Tax paid in advance Rs. 25,000 Cash Flows Plus Cash Flows Minus

39

16 17 18 19 20 21 22 23 24 25

Rent expenses incurred but not paid for Rs. 1 lakh Sold 3,000 bottles for cash Rs. 1 lakh Depreciation on fixed assets @ 20 % Commission income earned Rs.5 lakhs Interest expenses incurred @ 12% on ICICI loan Advertising expenditure incurred Rs. 0.5 lakhs. Rs. 0.25 lakhs has to be deferred. One customer Mr. X will not be able to pay (Rs. 20,000) and Management want a provision for doubtful debt @ 10% Purchased 10,000 bottles for Rs1 lakh and earned a cash discount of 10% Sold 10,000 bottles for Rs. 2 lakhs and recd. Rs. 1.8 lakhs in full satisfaction Provision for income tax @ 30 % TOTAL EXCESS OF CASH INFLOWS OVER CASH OUTFLOWS (as on 31.12.00) GRAND TOTAL

Answer to Assignment 35
Using common sense approach, list all the cash inflows and the cash outflows. Ascertain the closing cash position of M/s Aryan Traders Ltd., as on 31.12.2000 Transaction 1 2 Owner brings in equity share capital Rs. 10 lakhs Owner brings in preference share capital Rs. 2 lakhs Cash Flows Plus 10 2 Cash Flows Minus

40

Transaction 3 4 5 6 7 8 9 10 11 12 13 14 15 16 17 18 19 20 21 22 M/s Aryan traders Co. Ltd., borrowed on a long term basis from ICICI Rs. 4 lakhs @ 12% p.a Purchased land, building, plant & machinery for cash Rs.10 lakhs Purchased furniture & fixtures Rs.2 lakh on credit from M/s Z Purchase stock of 10,000 bottles of tablets @ Rs.10 per bottle on credit (M/s P) Purchased 5,000 bottles of tablets @ Rs.10 per bottle for cash. Returns to M/s P 1,000 bottles @ Rs.10 per bottle Sold for credit 6,000 @ Rs. 20 per bottle Sold for cash 4,000 bottles @ Rs. 20 per bottle Rent expenses incurred and paid Rs. 50,000 Paid insurance for two years Rs. 50,000 (Rs. 26,000 relates to 1990) Commission income received in cash Rs.25,000 Advances received from customers Rs. 1 lakh Tax paid in advance Rs.25,000 Rent expenses incurred but not paid for Rs. 1 lakh Sold 3,000 bottles for cash Rs. 1 lakh Depreciation on fixed assets @ 20% Commission income earned Rs. 5 lakh Interest expenses incurred @ 12% on ICICI loan Advertising expenditure incurred Rs. 0.5 lakh. Rs. 0.25 lakh has to be deferred One customer Mr. X will not be able to pay (Rs.20,000) and management wants a provision

Cash Flows Plus 4

Cash Flows Minus

10 nil nil nil nil 0.5 nil nil 0.8 nil nil 0.25 1 nil nil 1 nil nil nil nil nil nil nil nil 0.5 0.5 nil nil 0.25 nil nil nil nil nil nil nil

41

Transaction for doubtful debt @ 10% 23 24 25 Purchased 10,000 bottles for Rs. 1 lakh and earned a cash discount of 10% Sold 10,000 bottles for Rs. 2 lakhs and recorded Rs. 1.8 lakhs in full satisfaction Provision for income tax @ 30% TOTAL EXCESS OF CASH INFLOWS OVER CASH OUTFLOWS (as on 31.12.2000) GRAND TOTAL

Cash Flows Plus

Cash Flows Minus

nil 1.8 nil 20.85

0.9 nil nil 12.65 8.20

20.85

20.85

N.B. Excess of cash inflows over cash outflows represents the cash position as on 31.12.2000 and will appear as a liquid asset in the current asset section, on the asset side of the balance sheet as on 31.12.2000. The closing balance as on 31.12.2000 is the opening balance as on 01.01.2001.

Assignment 36
With the help of the financial statements of M/s Aryan Traders for the period ending 31st December 2000, fill in the following blanks in words and in rupees:

1. Gross Fixed assets as on 31.12.2000 = ____________________________________________________________


and equals Rs._______.

2. Net Fixed assets as on 31.12.2000 = ____________________________________________________________


and equals Rs._______.

3. Accumulated Depreciation as on 31.12.2000 = ____________________________________________________________


and equals Rs._______.

4. Current Assets as on 31.12.2000 = ____________________________________________________________


and equals Rs._______. Current assets are also known as Gross working capital.

5. The items of Current Assets as on 31.12.2000 include:

Rs.

42

Rs. Rs. Rs. Rs.

43

6. Current Liabilities as on 31.12.2000 = ____________________________________________________________


and equals Rs._______.

7. The items of Current Liabilities as on 31.12.2000 include:


8. 9.

Rs. Rs. Rs. Rs. Rs. Net Working Capital as on 31.12.2000 = ____________________________________________________________ and equals Rs._______. Quick Assets as on 31.12.2000 = ____________________________________________________________ and equals Rs._______. [Quick assets = current assets stock]

10. Long Term Liabilities as on 31.12.2000 = ___________________________________________________ and equals Rs._______. 11. Owners funds or Net worth as on 31.12.2000 = ____________________________________________________________and equals Rs. 12. Total Assets as on 31.12.2000 = ____________________________________________________________ and equals Rs._______. 13. Total sources as on 31.12.2000 =

Answer to Assignment 36
With the help of the financial statement of M/s Aryan Traders for the period ending 31st December 2000, fill in the following blanks in words and in rupees:

1. Gross fixed assets as on 31.12.2000 = Purchase price of fixed assets and equals Rs. 12 lakhs. 2. Net fixed assets as on 31.12.2000 = Gross fixed assets minus accumulated depreciation and equals
Rs. 12 lakhs Rs. 2.4 lakhs i.e., Rs.9.6 lakhs.

3. Accumulated depreciation as on 31.12.2000 = sum of the current depreciation charged year after year and equals
Rs. 2.4 lakhs. In the first year of the life of a fixed asset current depreciation and accumulated depreciation will be equal.

44

4. Current assets as on 31.12.2000 = Gross working capital and equals Rs. 14.96 lakhs. 5.The items of current assets as on 31.12.2000 include:
Stock Debtors Cash Prepaid insurance Advance tax Accrued income (Rs. Lakhs) 0.1 0.9 8.2 0.26 0.25 5.0 14.71

TOTAL

6) Current liabilities as on 31. 12. 2000 = short term sources and equals Rs. 4.68 lakhs

7) The items of current liabilities as on 31. 12. 2000 include:


Accounts payable (M/s P) Accounts payable (advertising) Income received in advance Outstanding expenses Interest expenses Provision for income tax TOTAL

(Rs.lakhs) 0.9 0.5 1.0 1.0 0.48 0.8 4.68

8) Net working capital as on 31. 12. 2000 = Current assets minus current liabilities and equals Rs.14.71 lakhs Rs. 4.68 lakhs i.e., Rs. 10.03 lakhs. 9) Quick assets as on 31. 12 2000 = those assets which are to be converted into cash very soon and is defined as current assets minus stock and equals Rs. 14.71 lakhs Rs. 0.1 lakhs i.e., Rs. 14.61 lakhs. 10) Long term liabilities as on 31. 12. 2000 = long term source and equals Rs. 6 lakhs. 11) Owned funds or net worth as on 31.12.2000 = owned sources of funds and equals Rs. 13.88 lakhs.

45

12) Total assets as on 31.12.2000 = Total uses of funds and equals Rs. 24.56 lakhs. 13) Total sources as on 31.12 .2000 = current liabilities (Rs. 4.68) lakhs + Long term Liabilities (Rs. 6.00) lakhs + Owned funds (Rs. 13.88) lakhs = Rs.24.56 lakhs.

Assignment 37
Present the balance sheet of M/s Aryan Traders Ltd., after all the 25 transactions as on 31.12.2000 in the two formats shown below.

Assignment 37 (format 1) Balance Sheet proforma - Horizontal Format


Liabilities & Capital Equity share capital Preference share capital Reserves Owners Funds Long Term liabilities Total Current Liabilities as per schedule 2 Total current assets as per schedule 1 Miscellaneous Assets Deferred advertising expenditure Total Liabilities and Capital Total Assets Amount Amount Gross Fixed assets Less: Accumulated depreciation Assets Amount Amount

NET FIXED ASSETS

46

BALANCE SHEET PRO FORMA (contd) Schedule 1 Current Assets

Amount

Amount

TOTAL
Schedule 2 Current Liabilities

Amount

Amount

TOTAL

47

Assignment 37 (Format 2)
Balance sheet pro forma - Vertical Format ASSETS: Gross Fixed Assets Less: Accumulated Depreciation Net fixed assets (A) Current assets (B)

Less: Current Liabilities (C)

Net Assets (A+B-C)

Balance sheet pro forma -Vertical Format (contd.)

48

FINANCED BY Long term Liabilities (D) Owners funds Equity share capital Preference share capital Reserves Total owners fund (E) Total financing (D+E) N.B

(1) Total financing is also known as capital employed. (2) Net Assets (A+B-C) should be equal to total financing (D+E).

49

Answer to Assignment 37 (Horizontal Format)


M/S Aryan Traders Limited Balance sheet as on 31.12.2000 Liabilities & capital Equity share capital Preference Share capital Reserves Owner's fund Long term liabilities Amount 10 2 1.88 13.88 6.00 Amount Gross fixed Assets Less: Accumulated depreciation NET FIXED ASSTES Assets Amount 12 (2.4) 9.6 Amount

Total current liabilities as per schedule 2

4.68 Total current assets as per schedule 1 Miscellaneous Assets: Deferred advertising expenditure

14.71 0.25

Total liabilities and capital

24.56 Total assets

24.56

50

Answer to Assignment 37.2


M/s Aryan Traders Ltd. (Vertical Format) Balance sheet as on 31.12.2000 Assets Fixed assets Less: Accumulated Depreciation Net Fixed assets (A) Current assets (B) Stock Debtors Less: bad debts Provision for doubtful debt Net debtors cash Prepaid Insurance Advance income tax Accrued income Miscellaneous assets Deferred advertising expenditure(C) Current liabilities (D) Accounts payable Account payable (advertising) Income received in advance Outstanding Expenses Interest expenses Provision for income tax 0.9 0.5 1.0 1.0 0.48 0.8 (4.68) 0.25 0.25 1.2 (0.2) (0.1) 0.9 8.2 0.26 0.25 5.00 14.71 0.1 12 (2.4) 9.6 Amount (Rs. lakhs)

51

M/s Aryan Traders Ltd. (Vertical Format) Balance sheet as on 31.12.2000 Assets Net Assets (A+B+C-D) Financed by Long term Liabilities (E) Owners' funds Equity share capital Preference share capital Reserves Total owners' fund (F) Total financing (E+F)

Amount (Rs. lakhs)

19.88

10 2 1.88 13.88 19.88

52

Assignment 38
Given the list of concepts below and the 25 transaction of M/s Aryan Traders Ltd., match the concepts with relevant transactions. Concept Transaction No. Money measurement Entity concept Going concern Accrual concept Matching concept Conservatism concept

Answer to Assignment 38
Given the list of concepts below and the transaction of M/s Aryan Traders Ltd., match the concepts with relevant transactions. Concept Money measurement Entity concept Going concern Accrual Concept Matching concept Conservatism concept Transaction no. All transaction All transactions All Transactions 9,12,14,16,19,20 9,12,14,16,18,19,20 22

53

Assignment 1 Rashid Enterprises


In the space provided below, for each action for Rashid Enterprises (REL), present the accounting equation, which reflects the transaction(s). At the end of action 12, draw up a balance sheet, profit and loss account and cash flow statement adopting the layouts given in the text. Action 1 Rashid commences business with Rs. 40,000 and a further Rs.10,000 cash from his cousin. The money received from his cousin is an interest free loan.

Action 2 REL buys the plant and equipment for Rs.5,000 by cash, factory and warehouse for Rs.25,000 by cash, and raw materials for Rs.8,000 (half by cash, half by credit).

Action 3 Before the equipment is commissioned, it requires a post installation lubrication costing Rs.200. REL pays for this in cash. 50 % of the raw materials are then processed into finished goods through the equipment and the labour cost is Rs.400. Action 4 REL pays his creditors in full and sells half of the finished goods (recorded at cost Rs.2, 200) for Rs.4, 000 credit

54

Action 5 REL buys second-hand equipment for Rs. 3,000 on credit and other assets for Rs.200 cash. Action 6 REL sells the remainder of the finished goods (recorded at cost of Rs.2,200) for Rs.3,900, and receives payment of Rs.3,900 from his debtors. Action 7 The equipment breaks down and requires Rs.100 for repairs, which REL pays in cash. REL buys further raw materials for Rs.6,000 cash and processes the remainder of his first batch of raw materials (which had cost Rs.4,000) at a cost for labour of Rs.300. Action 8 On Id, REL buys his wife a present costing Rs.100 and his secretary a gift costing Rs.1, 200. He pays for both items using his credit card.

Action 9 REL sells his second batch of finished goods (which are recorded at a cost of Rs.4,300) for Rs. 6,000, receiving half of the money in cash and giving credit for the other half. REL pays off his creditors.

Action 10 REL pays Rs.400 cash for advertising and Rs.200 cash for audit fees; REL also has all of his raw materials (cost Rs.6,000) processed, his labour force incurring Rs.1,000 wages in doing so.

55

Action 11 REL auditors advise that he should write off the debt of Rs.100 which has been outstanding since Action 4; in their opinion this debt is now irrecoverable. They also recommend that REL provide for depreciation on plant and equipment at a rate of 10 percent and on motor vehicles at 25 per cent.

Action 12 REL considers that one-fifth of his factory and warehouse space is excessive for his needs; he sells that part for Rs.7,000 in cash. He withdraws Rs.2,000 in cash for personal needs. Action 13 Income Tax @ 30%.

56

Solution to Assignment no. 1

RASHID ENTERPRISES The Equation following each Action [Action 1 to 13]


Action 1 Assets = Capital + Liabilities

Cash Rs. 50,000 = Owner's equity (OE) Rs. 40,000 + Long-term loan (LTL) Rs. 10,000. Action 2 Assets Plant and equipment Factory and warehouse Raw material inventory Cash Action 3 Assets Plant and equipment Factory and warehouse Raw material inventory Finished Goods inventory Cash = Rs. 5,200 Rs.25,000 Rs. 4,000 Rs. 4,400 Rs. 15,400 Capital OE Rs. 40,000 + Liabilities LTL Creditors = Rs. 5,000 Rs.25,000 Rs. 8,000 Rs. 16,000 Capital OE Rs. 40,000 + Liabilities LTL Creditors

Rs. 10,000 Rs. 4,000

Rs. 10,000 Rs. 4,000

57

Action 4 Assets Plant and equipment Rs. 5,200 Factory and warehouse Rs. 25,000 Raw material inventory Rs. 4,000 Finished Goods Inventory Rs. 2,200 Debtors Rs. 4,000 Cash Rs. 11,400 = Capital OE Rs. 41,800 + Liabilities LTL Rs. 10,000 Creditors Nil

Note that the post-installation lubrication has been 'capitalized'. We can gather from the action that the equipment would not work without this lubrication and so we can add this cost to the original purchase price. Any further maintenance on this equipment would be expensed', i.e., written off against owner's equity. Action 5 Assets Plant and equipment Factory and warehouse Motor vehicle Raw material inventory Finished goods inventory Debtors Cash = Rs. 5,400 Rs. 25,000 Rs. 3,000 Rs. 4,000 Rs. 2,200 Rs. 4,000 Rs. 11,200 Capital OE Rs. 41,800 + Liabilities LTL Rs. 10,000 Creditors Rs. 3,000

58

Action 6 Assets Plant and equipment Rs. 5,400 Factory and warehouse Rs. 25,000 Motor vehicle Rs. 3,000 Raw material inventory Rs. 4,000 Finished goods inventory Nil Debtors Rs. 100 Cash Rs. 19,000 Action 7 Assets Plant and equipment Factory and warehouse Motor vehicle Raw material inventory Finished goods inventory Debtors Cash = Rs.5,400 Rs. 25,000 Rs. 3,000 Rs. 6,000 Rs. 4,300 Rs. 100 Rs. 12,600 OE Capital Rs. 43,400 + Liabilities LTL Rs. 10,000 Creditors Rs. 3,000 = OE Rs Capital 43,500 + Liabilities LTL Rs.10,000 Creditors Rs. 3,000

Action 8 No change from Action 7. This action represents personal expenditure and does not affect Rashids business records.

59

Action 9

Assets Plant and equipment Rs. 5,400 Factory and warehouse s. 25,000 Motor vehicle Rs. 3,000 Raw material inventory Rs. 6,000 Finished goods inventory Nil Debtors Rs. 3,100 Cash Rs. 12,600

= OE

Capital Rs 45,100

Liabilities LTL Rs. 10,000 Creditors Nil

Action 10 Assets Plant and equipment Factory and warehouse Motor vehicle Raw material inventory Finished goods inventory Debtors Cash = Rs.5,400 Rs. 25,000 Rs. 3,000 Nil Rs. 7,000 Rs. 3,100 Rs. 11,000 OE Capital Rs. 44,500 + Liabilities LTL Rs. 10,000 Creditors Nil

60

Action 11 Assets Plant and equipment Factory and warehouse Motor vehicle Raw material inventory Finished goods inventory Debtors Cash Action 12 Assets Plant and equipment Rs. 4,860 Factory and warehouse Rs. 20,000 Motor vehicle Rs. 2,250 Raw material inventory Nil Finished goods inventory Rs. 7,000 Debtors Rs. 3,000 Cash Rs. 16,000 Action 13 Assets Plant and equipment Rs. 4,860 Factory and warehouse Rs. 20,000 Motor Vehicle Rs. 2,250 Raw material inventory Nil Finished goods inventory Rs. 7,000 Debtors Rs. 3,000 Cash Rs. 16,000 = OE Capital Rs. 41,577 + Liabilities LTL Rs. 10,000 Creditors Nil Provision for tax Rs.1,533 = OE Capital Rs. 43,110 + Liabilities LTL Rs. 10,000 Creditors Nil = Rs. 4,860 Rs. 25,000 Rs. 2,250 Nil Rs. 7,000 Rs. 3,100 Rs. 11,000 OE Capital Rs. 43,110 + Liabilities LTL Rs. 10,000 Creditors Nil

61

Profit and Loss Account for the period


Rs. Sales Less : Cost of sales : Raw Materials Labour Depreciation on plant and equipment Gross Manufacturing Profit Other Expenses: Motor repairs Advertising Depreciation on motor vehicles Bad debt Audit Operating Profit Profit from sales of factory Profit before tax Less: Provision for Income Tax @ 30% Profit after Tax 8,000 700 540 100 400 750 100 200 Rs. 13,900 9,240 4,660

1,550 3,110 2,000 5,110 1,533 3,577

62

Balance Sheet as at the end of Action 13


Rs. Fixed Assets Factory and warehouse Plant and equipment Motor vehicles Current Assets Finished goods Debtors Cash Less: Current Liabilities (Provision for Income tax) Net assets of the company Represented by: Capital introduced Profit after tax Less: Drawings Owners equity Long-term loan Rs. 20,000 4,860 2,250 Rs.

27,110

7,000 3,000 16,000

26,000 (1,533)

24,467 51,577

40,000 3,577 43,577 2,000 41,577 10,000

51,577

63

Cash Flow Statement for the period to action 13


Rs. Sources of cash Profit from operations Adjustment for non-cash items depreciation Capital introduction Long-term loan Sale of factory and warehouse Uses of cash Purchase of assets: Factory & warehouse Plant & equipment Motor vehicles Increase in inventories Finished goods Increase in debtors Drawings Closing balance of cash 3,110 1,290 40,000 10,000 Rs. 4,400 50,000 7,000 61,400

25,000 5,400 3,000 7,000 3,000

33,400 10,000 2,000 45,400 16,000

64

Assignment 2 Thomas Sons Thomas Sons opened their business for trading on 1 Jan. 2003 with Rs.25,000 cash. The first six months of trading resulted in the following transactions: Action 1 Action 2 Action 3 Action 4 Paid six months rent of Rs.2, 000 for the premises. Purchased equipment for Rs.10, 000 and an estate car for Rs.6, 000. Acquired Rs.8000 of manufacturing materials on credit, half of which was paid in June. Paid Rs.2000 in manufacturing wages in converting 75 per cent of the materials into finished goods.

Action 5

Sold 60 per cent of the finished products for Rs. 12,000 cash. . Paid Rs. 600 for office staff wages and 300 for petrol.

Action 6

Further information The equipment is estimated to have a useful life of five years, whilst the estate car requires to be depreciated over three years. Required: 1. 2. Prepare the accounting equations after each of the above actions; include depreciation too along with action (6). Prepare a profit and loss account for the six months to 30 June and a balance sheet as at that date after taking into account an income tax rate of 30% (Consider income tax adjustment as Action7).

65

Solution to Assignment no. 2

Thomas Sons
The initial accounting equation is: Assets (cash) Rs.25,000 = Owners Funds Rs.25,000. Thereafter, the accounting equation changes after each economic action as shown below: (a) Payment of rent reduces both cash and owners funds, since it is a charged to profit and loss account as an expense. Rs. Assets = Liability + Capital Cash = Rs. 23,000 OF = Rs. 23,000

(b) The equipment and car are acquisition of fixed assets, with no immediate effect on owners equity, but a decrease in cash. Rs. Assets = Liability + Capital Cash = Rs.7,000 OF = Rs. 23,000 Equipment = Rs.10,000 Vehicle = Rs. 6,000

(c) This increases the assets by introducing raw material inventory (Rs.8,000), financed by creditors-liabilities, half of which (Rs. 4,000) was paid by the end of June, depleting the cash amount. Rs. Assets = Liability + Capital Cash = Rs. 3,000 Creditors = Rs. 4,000 OF = Rs 23,000 Equipment = Rs.10,000 Vehicle = Rs.6,000 Raw material inventories = Rs.8,000

66

(d) Payment of manufacturing wages (Rs. 2,000) is expected to add value to the inventories, for recovery by future sale of the finished goods. Rs. Assets = Liability + Capital Cash = Rs. 1,000 Creditors = Rs. 4,000 OF = Rs. 23,000 Equipment = Rs. 10,000 Vehicle = Rs. 6,000 Raw material inventories = Rs. 2,000 Finished goods inventories=Rs.8,000 (RM = 6,000 + Wages = 2,000) (e) There are no two separate effects on the accounting equation from this business transaction. Sales impact owners equity, less the costs incurred in generating the sales. The amount of cash rises, but a debtor is created to reflect the balance outstanding of unpaid sales. Rs. Assets = Liability + Capital Cash = Rs. 8,500 (1,000+7,500) Creditors = Rs. 4,000 OF = Rs. 30,200 Equipment = Rs. 10,000 (23,000 + 7,200) Vehicle = Rs. 6,000 Raw material inventories = Rs. 2,000 Finished goods inventories = Rs. 3,200 (8,000 4,800) Debtors = Rs. 4,500 (12,000 7,500) (f) Both payments are administrative expenses, comprising part of the operating the business in the six months. Therefore, their effect is to reduce both cash and owners equity. Rs. Assets = Liability + Capital Cash = Rs. 8,500 Creditors = Rs. 4,000 OF= Rs. 27,300 Equipment =Rs. 9,000 (10,000 1,000) Vehicle = Rs. 5,000 (6,000 1,000) Raw material inventories = Rs. 2,000 Finished goods inventories = Rs. 3,200 Debtors =Rs. 4,500

67

Depreciation on equipment and vehicles is treated differently; the former being an element in the manufacturing process and the latter a selling/administrative expense. Even with the different lifetime for each assets type, the annual depreciation for each is Rs.2,000; reducing to Rs.1,000 each for the six months. (g) Provision for income tax. Assets = Liability Cash = Rs. 8,500 Creditors = Rs. 4,000 Equipment = Rs. 9,000 (10,000 1,000) Provision of income tax Vehicle = Rs. 5,000 (6,000 1,000) = Rs. 690 Raw material inventories =Rs. 2,000 Finished goods inventories = Rs. 3,200 Debtors = Rs. 4,500 + Capital OF= Rs. 26,610

68

Profit and loss account for the six months ended 30th June 2003. Rs. Sales Less: Cost of sales Materials Labour Gross manufacturing profit Less: Selling and administrative costs Rent Office wages Petrol Depreciation vehicle Profit before tax Less: Provision for income tax @ 30% 2,000 600 300 1,000 3,600 1,200 1,000 Rs. 12,000 5,800 6,200

3,900 2,300 (690) 1610

69

Balance sheet as at 30th June 2003. Assets Fixed assets Equipment at cost Less: Depreciation Vehicle at cost Less: Depreciation Current assets Stocks Debtors Cash Less: Current liabilities Creditors Provision for income tax Rs. 10,000 1,000 6,000 1,000 5,200 4,500 7,600 Rs. 9,000 5,000 14,000 Rs.

17,300 4,000 690 12,610 26,610

Represented by Capital introduced Reserves and surplus

25,000 1,610 26,610

70

Assignment 3 M/s X Co. Ltd. M/s X Co. Ltd., has the following transactions for the period ending 31.03.2002. Rs. in lakhs Date 1.1.2002 1.1.2002 1.1.2002 1.2.2002 1.2.2002 1.2.2002 1.2.2002 15.3.2002 15.4.2002 16.5.2002 17.6.2002 20.6.2002 28.12.2002 30.12.2002 31.12.2002 Particulars Equity Share Capital contributed by shareholders Preference Share Capital contributed by shareholders Procured loan from ICICI @ 10% per annum Purchased fixed assets Purchased I.T. equipment in cash Purchases Purchase returns Sales Sales returns Sales of Rs.10 lakhs received cash after providing discount of 2% Received cash from customers Rs. 5.9 lakhs in full satisfaction of Rs. 6 lakhs Payment to suppliers Rs. 2.8 lakhs in full satisfaction of Rs. 3 lakhs Rent Income received in cash Commission Income received in cash Rent expenses paid 6 5 1 Amount 50 10 20 40 5 6 1 10 2

71

Insurance expenses paid 31.12.2002 Advertising expenditure

2.5 3

Part 1 Journalize the above transactions Post them into the ledger Prepare a trial balance as on 31.12.2002 Part 2 You are provided with the following information on 31.12.2002 21) Closing stock is valued at Rs.1.5 lakhs. The market price as on 31.3.2002 is Rs.1.25 lakhs. 22) Outstanding rent expenses Rs.0.5 lakhs 23) Insurance is prepaid to the extent of Rs.0.75 lakhs 24) Commission Income accrued Rs.4 lakhs 25) Rent income is pre-received to the extent of Rs.3 lakhs 26) Advertising expenditure is to be spread over 3 years 27) Depreciation is to be provided @ 10% of the fixed assets and 25% of the I.T. equipment 28) Bad debts Rs. 50,000 29) Provision for doubtful debts @ 10% 30) Provision for Income tax @ 30% 31) Provision for dividends @ 20% of profits 32) Residual reserves (if any) to be transferred to Staff Welfare Reserve and General Reserve on a fifty-fifty basis. Required: Prepare the relevant financial statements viz. Income and other statements and Balance sheet for the year 2002.

72

Solution to Assignment no 3 Part 1

Journal Entries In the books of M/s X Company Ltd. (Rs. in lakhs) Amount (Debit) Dr 50 50 Dr 10 10 Dr 20 20 Dr 40 40 Dr 5 5 Dr 6 6 Dr 1 1 Dr 10 Amount (Credit)

Date 1. 1. 2002

Particulars Cash A/c To Equity share capital A/c

1. 1.2002

Cash A/c To Preference share capital

31. 1. 2002

Cash A/c To ICICI loan A/c

1. 2. 2002

Fixed assets A/c To Fixed assets loans A/c

1. 2. 2002

IT equipment A/c To Cash A/c

1. 2. 2002

Purchases A/c To Sundry Creditors A/c

1. 2. 2002

Sundry Creditors A/c To Purchase Return A/c

15. 3. 2002

Sundry Debtors A/c

73

To Sales A/c 15. 4. 2002 Sales returns A/c To Sundry Debtors A/c 16. 5. 2002 Cash A/c Sales Discount A/c To Sales A/c 17. 6. 2002 Cash A/c Sales Discount A/c To Sundry Debtors A/c 20. 6. 2002 Sundry Creditors A/c To Cash A/c To Purchase discount A/c 28. 12. 2002 Cash A/c To Rent Income 30. 12. 2002 Cash A/c To Commission Income 31. 12. 2002 Rent Expense A/c Insurance Expense A/c To Cash A/c Dr 1 Dr 2.5 Dr 5 Dr 6 Dr 3 Dr 5.9 Dr 0.1 Dr 9.8 Dr 0.2 Dr 2

10

10

2.8 0.2

3.5

74

31. 12. 2002

Advertising A/c To Sundry Creditors (Advertising) A/c

Dr 3 3

In the books of M/s X Co. Ltd., Ledger Accounts Dr Date 1. 1. 2002 Particulars To equity share capital To preference share capital To ICICI loan To cash sales To receipt from debtors To rent income To commission income Cash/ bank A/c Amount 50 Date 1. 2. 2002 Particulars By IT Equipment By payment to creditors By rent expense By insurance expense By balance c/d Cr Amount 5

1. 1. 2002 31. 1. 2002 16. 5. 2002 17. 6. 2002 28. 12. 2002 30. 12. 2002

10 20 9.8 5.9 6 5 106.7

20. 6. 2002 31. 12. 2002 31. 12. 2002 31. 12. 2002

2.8 1 2.5 95.4

106.7

75

Dr Date 31. 12. 2002 Particulars

Equity Share Capital A/c Amount 50 50 Date 1. 1. 2002 Particulars By cash Amount 50 50 To balance c/d

Cr

Dr Date 31. 12. 2002 Particulars

Preference Share Capital A/c Amount 10 10 Date 1. 1. 2002 Particulars By cash Amount 10 10

Cr

To balance c/d

Dr Date 31. 12. 2002 Particulars To balance c/d

ICICI Loan A/c Amount 20 20 Date 31. 1. 2002 Particulars By cash Amount 20 20

Cr

Dr Date 1. 2. 2002 1. 2. 2002 Particulars To fixed assets loan A/c To Cash (IT equipment)

Fixed Assets A/c Amount 40 5 45 45 Date 31. 1. 2002 Particulars By cash

Cr Amount 45

76

Dr Date 31. 12. 2002 Particulars To balance c/d

Fixed Assets Loan A/c Amount 40 40 Date 1. 2. 2002 Particulars By fixed assets A/c

Cr Amount 40 40

Dr Date 1. 2. 2002 Particulars To creditors

Purchase A/c Amount 6 6 Date 31. 12. 2002 Particulars By balance c/d

Cr Amount 6 6

Dr Date 1. 2. 2002 20. 6. 2002 20. 6. 2002 31. 12. 2002

Sundry Creditors A/c Particulars To purchase return A/c To cash A/c To purchase discount A/c To balance c/d Amount 1 2.8 0.2 2 6 6 Date 1. 2. 2002 Particulars By purchases A/c

Cr Amount 6

77

Dr Date 31. 12. 2002 Particulars

Purchase Return A/c Amount 1 1 Date Particulars

Cr Amount 1 1

To balance c/d

1. 2. 2002 By sundry creditors

Dr Date 15. 3. 2002 Particulars To sales

Sundry Debtors A/c Amount 10 Date 15. 4. 2002 16. 5. 2002 17. 6. 2002 31. 12. 2002 10 Particulars By sales return a/c By cash a/c By sales discount By balance c/d Amount 2 5.9 0.1 2 10

Cr

Dr Date 31. 12. 2002 Particulars To balance c/d

Sales A/c Amount 20 Date 15. 3. 2002 16. 5. 2002 16. 5. 2002 20 Particulars 10 By cash a/c By sales discount

Cr Amount 2 9.8 0.2 20

78

Dr Date 15. 4. 200 2 Particulars

Sales Return A/c Amount 2 2 Date 31. 12. 2002 Particulars By balance c/d

Cr Amount 2 2

To sundry debtors a/c

Dr Date 16. 5. 200 2 17. 6. 200 2 Particulars To sales a/c

Sales Discount A/c Amount 0.2 0.1 0.3 0.3 2 Date 31. 12. 2002 Particulars By balance c/d

Cr Amount 0.3

To sundry debtors a/c

Dr Date Particulars

Purchase Discount A/c Amount 0.2 0.2 Date 20. 6. 2002 Particulars

Cr Amount

15. 4. 2002 To balance c/d

By sundry creditors 0.2 a/c 0.2

79

Dr Date 31. 12. 2002 Particulars To cash a/c

Rent Expense A/c Amount 1 1 Date 31. 12. 2002 Particulars By balance c/d

Cr Amount 1 1

Dr Date 31. 12. 2002

Insurance Expense A/c Particulars To cash a/c Amount 2.5 2.5 Date 31. 12. 2002 Particulars By balance c/d

Cr Amount 2.5 2.5

Dr Date 30. 12. 2002

Commission Income A/c Particulars To balance c/d Amount 5 5 Date 30. 12. 2002 By cash a/c

Cr Amount 5 5

Dr Date 28. 12. 2002 Particulars To balance c/d

Rent Income A/c Amount 6 6 28. 12. 2002 Particulars By cash/ bank

Cr Amount 6 6

80

Dr Date

Advertisement Expenditure A/c Particulars Amount 3 Date 31. 12. 2002 Particulars

Cr Amount 3

31. 12. 2002 To Advertising By Balance c/d (Accounts Payable) A/c

Dr Date 31. 12. 2002 Particulars

Advertising Accounts Payable A/c Amount 3 Date 31. 12. 2002 Particulars By Advertising (accounts payable) A/c

Cr Amount 3

To balance c/d

81

M/s X Co. Ltd. Trial balance as on 31 December 2001 (Amount in Rs. lakh) Particulars Equity share capital Preference share capital ICICI loan @10% Fixed assets Purchases Purchase returns Sales Sales return Sales discount Purchase discount Rent expenses Insurance expenses Commission income Rent income Advertising expenditure 3 1 2.5 5 6 2 0.3 0.2 45 6 1 20 Debit Credit 50 10 20

82

Fixed assets (loan) Advertising (accounts payable) Cash Sundry Debtors Sundry Creditors Total 157.2 95.4 2

40 3

2 157.2

83

M/s X Co. Ltd.


Income Statement for the year ended 31 December 2001 Particulars Net sales Less: Trading Cost of Goods Sold Gross Trading Profit Less: Non Trading expenses Operating profit Add: income other than sales Profit before interest and tax (P.B.I.T) Less: interest @ 10% on ICICI loan Profit before tax (P.B.T) Less: provision for tax @30% Profit after tax (P.A.T) 4 3 1 2 (Rs. in lakhs) Schedule Amount 17.7 (3.55) 14.15 (10.15) 4.00 12.00 16.00 2.00 14.00 4.20 9.80

84

Schedule showing appropriations


Rs. lakh Balance as per income statement (P.A.T) Add: balance from earlier years Available for appropriations Less: provision for dividends @ 20% (rounded off) Balance Less: Transfer to Staff Welfare Reserves Less: Transfer to General Reserves 9.80 nil 9.80 2.00 7.80 3.90 3.90

85

M/s X Co. Ltd Balance sheet as on 31 December 2001 Rs. lakh Liabilities Owners funds Equity share capital 50 Rs. Assets Net fixed assets Current assets (per schedule 5) Miscellaneous assets (to the extent not written off ) Deferred Advertising Reserves/ surplus Long term liabilities ICICI loan Fixed assets (vendors) Current liabilities (per schedule 6) 20 40 60 16.70 144.50 144.50 7.80 67.80 Rs. 39.75 102.75 2.00

Preference share. capital 10

86

Schedule 1 (showing net sales) Sales Less: sales return

Rs. lakh 20.00 2.00 18.00

Less: sales discount Net sales Schedule 2 (showing trading cost of goods sold)

0.30 17.70

Rs. lakh Opening stock Purchases Less: purchase returns Nil 6.00 (1.00) 5.00 Less: purchase discount (0.20) 4.80 Less: closing stock ( lower of cost or market value) Cost of goods sold (COGS) 1.25 3.55

87

Schedule 3 (showing non-trading expenses) Rs lakh Rent Add: Outstanding rent 1.00 0.50 1.50 Insurance Less: Prepaid Insurance 2.50 0.75 1.75 Depreciation (schedule 3 A) Bad debts Add: provision for doubtful debts (10% of 1.5) 0.50 0.15 0.65 1 10.15 5.25

Advertising expenditure (1/3 to be charged) / and balance to be deferred Total

Schedule 3 A (depreciation)
Rs. lakh Particulars (gross block) Fixed assets @ 10% I.T equipment 40.00 5.00 Amount Current depreciation 4.00 1.25 Accumulated depreciation 4.00 1.25 Net fixed assets

36.00 3.75

88

@ 25% Total 45.00 5.25 5.25 39.75

Schedule 4 (showing income other than sales) Rs. lakh Rent income Less: Pre-received rent income Commission Income Add: Accrued Commission Income Total Schedule 5: Showing details of current assets Rs. lakh Cash Debtors Stock Pre-paid insurance Commission accrued 95.40 1.35 1.25 0.75 4.00 Total current assets 102.75 Schedule 6: Showing details of current liabilities Rs. lakh 6.00 (3.00) 5.00 4.00 9.00 12.00 3.00

89

Creditors Pre-received rent Outstanding rent Outstanding interest Advertisement expenses (accounts payable) Provision for tax Proposed dividend Total current liabilities

2.00 3.00 0.50 2.00 3.00 4.20 2.00 16.70

90

Assignment 4 M/s SPJ & Co. Ltd M/s SPJ & Co. Ltd., has the following transactions for the year-ended 31.12.2001 (Amount in Rs. lakhs) 1.1.2001 1.1.2001 1.1.2001 15.2.2001 17.2.2001 22.2.2001 27.2.2001 17.3.2001 16.4.2001 18.5.2001 11.6.2001 20.6.2001 25.12.2001 Equity Share Capital contributed by shareholders Preference Share Capital contributed by shareholders Procured loan from IDBI @ 10 % p.a. Purchased fixed assets on account Purchased I.T. equipment in cash Purchases on account Purchase returns Sales on account Sales returns Sales of Rs. 10 lakhs - Received cash after providing discount of 10 % Received cash from customers Rs. 5 lakhs in full satisfaction of Rs. 6 lakhs Payments to suppliers Rs. 2 lakhs in full satisfaction of Rs. 3 lakhs Rent income received in cash 8 80 20 40 30 10 6 1 10 2

91

31.12.2001 31122001 31.12.2001

Rent expenses paid Insurance expense paid Commission income received in cash Advertising expenditure accrued

2 4 7 4

On the basis of data given above: 5. Journalize the transactions 6. Post the transactions into the Ledger 7. Prepare a Trial balance as on 31.12 2001. You may use the following assumptions for solving the given problem: Ignore pro rata rates for the purpose of depreciation and interest computations; In transaction dated 11.6.2001, the difference be treated as discount allowed; In transaction dated 20.6.2001, the difference be treated as discount received ADJUSTMENTS It is now 31.12.2001 You are provided with the following information: 1. Closing Stock is valued at Rs. 2 lakhs. The market price as on 31.12 2001 is Rs.20 lakhs. 2. Bad Debts Rs. 0.5 lakh. 3. Advertising Expenditure is to be spread over 4 years. 4. Outstanding Rent Expenses Rs. 25 lakhs. 5. Commission Income Accrued Rs. 0.95 lakh. 6. Insurance is prepaid to the tune of Rs.1 lakh.

92

7. Provision for Doubtful Debts @ 10 % 8. Depreciation is to be provided @ 10 % of the fixed assets and 25 % of I.T Equipment 9. Rent income is pre-received to the extent of Rs 4 lakhs 10. Provision for I. Tax @ 30 % (ignore surcharge) 11. Provision for Dividends @ 20 % of profits 12. Residual Reserves (if any) to be transferred to Staff Welfare Reserve and General Reserve on a 40:60 basis. Required: 1. Prepare the relevant financial statement: Income and other statements and Balance sheet for the year 2001.

Solution to Assignment no. 4 M/s SPJ

Journal entries Date 1.1.2001 Particulars Cash/ Bank A/c To Equity share capital A/c (being amount received on issue of equity shares) 1.1.2001 Cash/ Bank A/c To Preference share capital A/c (being amount received on issue of preference shares) Dr 20 20 Dr L/F Debit (Rs.) 80 80 Credit (Rs.)

93

1.1.2001

Cash/ Bank A/c To Loan from IDBI A/c (being loan taken from IDBI)

Dr

40 40

15.2.2001

Fixed assets A/c To Fixed assets loan A/c (being fixed assets purchased on account)

Dr

30 30

17.2.2001

IT equipment A/c To Cash A/c (being equipment purchased and paid for in cash)

Dr

10 10

22.2.2001

Purchases A/c To Sundry Creditors A/c (being purchases made on account)

Dr

6 6

27.2.2001

Sundry Creditors A/c To purchase return A/c (being goods returned on purchase)

Dr

1 1

17.3.2001

Sundry Debtors A/c To sales A/c (being goods sold)

Dr

10 10

94

16.4.2001

Sales return A/c To Sundry Debtors A/c (being sales return on goods sold)

Dr

2 2

18.5.2001

Cash/ Bank A/c Discount allowed A/c To sales A/c (being payment recd. from debtors after allowing discount@10%)

Dr Dr

9 1 10

11.6.2001

Cash/ Bank A/c Discount allowed A/c To Sundry debtors A/c (being amount recd. in full and final settlement from debtors)

Dr Dr

5 1 6

20.6.2001

Creditors A/c To Cash A/c To discount recd A/c (being amount paid to suppliers on full satisfaction of their claims)

Dr

3 2 1

95

25.12.2001

Cash/ Bank A/c To rent income A/c (being income recd. as rent)

Dr

8 8

31.12.2001

Rent expenses A/c Insurance expense A/c To Cash A/c (being expenses paid in cash)

Dr Dr

2 4 6

31.12.2001

Cash/ Bank A/c To commission income A/c (being commission income recd)

Dr

7 7

31.12.2001

Advt. Expense A/c To Advt. Accounts payable A/c (being advt. expenses accrued but not yet paid

Dr

4 4

96

In Books of M/s SPJ Co. Ltd. Ledger Accounts

Cash/ Bank
Dr To equity share capital To pref. share capital To loan from IDBI To sales (cash sales) To debtors To rent income To commission income 80 20 40 9 5 8 7 169 169 Bu balance c/d 151 By IT equipment By creditors By rent expenses By insurance expenses 10 2 2 4 Cr

Equity share capital A/c


Dr To balance c/d 80 By Cash/ Bank 80 Cr

80

80

97

Preference share capital A/c


Dr To balance c/d 20 By Cash/ Bank 20 Cr

20

20

IDBI loan A/c


Dr To balance c/d 40 By Cash/ Bank 40 Cr

40

40

Fixed assets A/c


Dr To fixed assets loan A/c 30 30 By balance c/d 30 30 Cr

IT equipment A/c
Dr To cash/ bank 10 By balance c/d 10 Cr

10

10

98

Fixed assets loan A/c


Dr To balance c/d 30 By fixed assets A/c 30 Cr

30

30

Purchases A/c
Dr To creditors 6 By balance c/d 6 Cr

Creditors A/c
Dr To purchase return To cash/ bank To discount recd. To balance c/d 1 2 1 2 6 6 By purchases 6 Cr

99

Purchase return A/c


Dr To balance c/d 1 By creditors 1 Cr

Sundry Debtors A/c


Dr To sales 10 By sales return By cash/ bank By discount allowed By balance c/d 10 2 5 1 2 10 Cr

Sales A/c
Dr To balance c/d 20 By debtors By cash/bank A/c 20 10 10 20 Cr

100

Sales return A/c


Dr To debtors 2 By balance c/d 2 2 2 Cr

Discount allowed A/c


Dr To sales To debtors 1 1 2 By balance c/d 2 2 Cr

Discount received A/c


Dr To balance c/d 1 By creditors 1 Cr

Rent income A/c


Dr To balance c/d 8 By cash/ bank 8 Cr

101

Rent expenses A/c


Dr To cash/ bank 2 By balance c/d 2 2 2 Cr

Insurance expenses A/c


Dr To cash/ bank 4 4 By balance c/d Cr 4 4

Advertisement expenses A/c


Dr To advt. accounts payable 4 By balance c/d 4 4 4 Cr

Commission income A/c


Dr To balance c/d 8 1 7 7 By cash/ bank Cr 7

102

Advt. Accounts payable A/c


Dr To balance c/d 4 By advt. expenses Cr 4

4 In the Books of M/s SPJ Co. Ltd. Trial balance as at 31. 12. 2001 Particulars Cash/ bank Equity share capital Preference share capital 10% IDBI loan Fixed assets IT equipment Fixed assets loan Purchases Creditors Purchase return Debtors 2 6 2 1 30 10 30 Debit (Rs.) 151 80 20 40

Credit (Rs.)

103

Sales Sales return Discount allowed Discount received Rent income Rent expense Insurance expense Commission income Advertisement expense Advt. Accounts payable Total 213 4 2 4 2 2

20

1 8

4 213

104

Income Statement for the year ended 31 December 2001 (Amount in Rs. lakh) Particulars Net sales Less: Trading Cost of Goods Sold Gross Trading Profit Less: Non-trading expenses Operating profit Add: income other than sales Profit before interest and tax (P.B.I.T) Less: interest @ 10% on IDBI loan Profit before tax (P.B.T) Less: provision for tax @30% Profit after tax (P.A.T) 4 3 Schedule 1 2 Amount 16.00 (2.00) 14.00 (12.40) 1.60 11.95 13.55 (4.00) 9.55 (2.865) 6.865

105

Schedule showing appropriations (Amount in Rs. lakh) Balance as per income statement (P.A.T) Add: balance from earlier years Available for appropriations Less: provision for dividends @ 20% (rounded off) Balance Less: Transfer to Staff Welfare Reserves Less: Transfer to General Reserves 6.865 Nil 6.865 (1.373) 5.492 2.196 3.295

106

Balance sheet as on 31 December 2001 (Amount in Rs. lakh) Liabilities Owners funds Equity share capita Preference share capital Reserves/ surplus Long term liabilities IDBI loan Fixed assets (loans) Current liabilities 40 30 70 18.55 193.80 193.80 80 20 Rs. Assets Net fixed assets Current assets Misc. assets (deferred adv. Expenditure) 5.25 105.25 Rs. 34.50 156.30 3.00

107

Details of current assets (Amount in Rs. lakh) Cash Debtors Stock Pre paid insurance Commission accrued Total current assets 151.00 1.35 2.00 1.00 0.95 156.30

Details of current liabilities (Amount in Rs. lakh) Creditors Pre received rent Outstanding rent Outstanding interest Advertisement expenses accounts payable Provision for tax Proposed dividend Total current liabilities 2.00 4.00 0.25 4.00 4.00 3.00 1.30 18.55

108

Schedule 1 (showing net sales) (Amount in Rs. lakh) Sales Less: sales return 20.00 (2.00) 18.00 Less: sales discount Net sales Schedule 2 (showing trading cost of goods sold) (Amount in Rs. lakh) Opening stock Purchases Less: purchase return Nil 6.00 (1.00) 5.00 Less: purchase discount (1.00) 4.00 Less: closing stock (lower of cost or market value) Cost of goods sold (COGS) (2.00) 2.00 (2.00) 16.00

109

Schedule 3 (showing non trading expenses) (Amount in Rs. lakh) Rent Add: Outstanding rent 2.00 0.25 2.25

Insurance Less: Prepaid

4.00 (1.00) 3.00

Depreciation (schedule 3 A) Bad debts Add: Provision for doubtful debts (10% of 1.5)

5.5 0.50 0.15 0.65

Advertising expenditure (1/3 to be charged) Total

1 12.40

110

Schedule 3 A (depreciation) (Amount in Rs. lakh) Particulars (gross block) Fixed assets @ 10% I.T equipment @ 25% 30.00 10.00 Amount Current depreciation 3.00 2.50 Accumulated depreciation 3.00 2.50 Net fixed assets

27.00 7.50

Totals

40.00

5.50

5.50

34.50

Schedule 4 (showing income other than sales) (Amount in Rs. lakh) Commission income Add: Accrued commission Rent income Less: Pre-received Total 7.00 0.95 8.00 (4.00) 4.00 11.95 7.95

111

Assignment 5 M/s Dalvi Co. Ltd. A) M/s Dalvi Co. Ltd. enters into the following transactions for the year ended 31.12.2002
Issued Issued Issued

equity shares worth Rs. 1,00,000. 500 (12%) preference shares of Rs.100 at Rs.110. Preferential dividend paid in cash on 31.12.2002. 9% debentures worth Rs. 1,00,000 at Rs. 95,000. Interest on debentures is paid in full on 31.12.2002. goods on credit worth Rs. 10,000.

Purchased Of

the above, goods worth Rs.2,000 were not up to specifications and were hence returned. following expenses were paid for in cash Rent Rs. 2,000, Insurance Rs. 3,000, Advertisement Rs. 5,000 commission for Rs. 6,000.

The

Received Rent Paid

income received during the year was Rs. 4,000. Sundry Creditors Rs. 4,000 in full satisfaction of Rs. 4,200 Rs. 20,000 on credit Rs. 5,000 for cash and obtained Rs. 4000 in full satisfaction.

Sales Sales

Part 1 Required: Journalize the above transactions, prepare the ledger accounts and a trial balance.

112

B) Part 2 It is now 31.12.2002 With the help of the following adjustments prepare the Income statement and Balance sheet.

Adjustments:
1.Closing Stock is valued at cost Rs. 1,500. The market price is Rs. 1,200. 2.Outstanding rent expenses Rs. 1,000. 3.Prepaid insurance expensesRs.1,400. 4.Accrued Commission income Rs. 10,000. 5.Pre-received rent income Rs. 1,500 . 6.Bad debts Rs. 2,000. 7.Provision for doubtful debts @ 10%. 8.Advertising expenditure to be spread over 3 years in the ratio of 2:2:1 respectively. 9.Discount on the issue of debentures is to be written off against profit and loss account. 10.Provision for income tax @ 30%. 11.Provision for dividends @ 50% of PAT available for equity shareholders. 12.Balance is to be held as General Reserves (50%) and Surplus (50%).

113

Solution to Assignment no. 5 Part 1 In the Books of M/s Dalvi Co. Ltd.
Journal entries Date 1 Particulars Cash A/c To equity share capital 2 Cash A/c To 12% preference share capital To share premium 3 Cash A/c Discount on issue of debentures a/c To 9% debentures a/c 4 Purchase a/c To sundry creditors 5 Sundry creditors A/c To purchase returns A/c Dr 2,000 2,000 Dr 10,000 10,000 Dr 95,000 5,000 1,00,000 Dr 55,000 50,000 5,000 L/F Debit (Rs.) 1,00,000 1,00,000 Credit (Rs.)

114

Rs. 6 Rent expenses a/c Insurance expenses a/c Advertisement a/c To cash a/c 7 Cash A/c To commission income A/c 8 Cash/ Bank A/c To rent income 9 Sundry Creditors a/c To cash a/c To purchase discount a/c 10 Cash a/c To sales 11 Cash a/c Sales discount a/c To sales a/c 12 Interest on debentures A/c To cash a/c Dr 9,000 9,000 Dr 4000 1000 5000 Dr 20000 20000 Dr 4200 4000 200 Dr 4,000 4,000 Dr 6,000 6,000 Dr 2,000 3,000 5,000 10,000

Rs.

115

13

Preference dividend A/c To cash / bank

Dr

6,000 6,000

116

Ledger Accounts Cash/ Bank Dr To equity share capital To 12% pref. share capital To 9% debentures To share premium To rent income To commission income To Cash sales 1,00,000 By rent expenses 50,000 By insurance expenses 95,000 By advertising expenses 5,000 By dividend on preference shares 4,000 By interest on debentures 6,000 By sundry creditors 4,000 By balance c/d 2,64,000 Cr 2,000 3,000 5,000 6,000 9,000 4,000 2,35,000 2,64,000

Equity share capital A/c Dr To balance c/d 1,00,000 1,00,000 By Cash A/c 1,00,000 1,00,000 Cr

117

12% Preference share capital A/c Dr To balance c/d 50,000 By cash A/c 50,000 Cr

50,000 Share premium A/c Dr To balance c/d 5,000 By cash A/c

50,000

Cr 5,000

5,000 Purchases A/c Dr To Sundry Creditors 10,000 10,000 Sundry Creditors A/c Dr To purchase return a/c To cash A/c To purchase discount To balance c/d 2,000 By purchases 4,000 200 3,800 10,000 By balance c/d

5,000

Cr 10,000 10,000

Cr 10,000

10,000

118

Purchase return A/c Dr To balance c/d 2,000 By Sundry creditors 2,000 Cr.

2,000

2,000

Preferential dividend A/c Dr To cash a/c 6,000 By balance c/d 6,000 Discount on issue of debentures A/c Dr To 9% debentures 5,000 By balance c/d 5,000 9% debentures A/c Dr To balance c/d 1,00,000 By cash / bank By disc. on issue 1,00,000 95,000 5,000 1,00,000 Cr. 5,000 5,000 Cr. 6,000 6,000 Cr.

119

Interest on debentures A/c Dr To cash / bank 9,000 By balance b/d 9,000 Cr.

9,000 Sales A/c Dr To balance c/d 25,000 By sundry debtors A/c By cash A/c By sales discount A/c 25,000 Sundry Debtors A/c Dr To Sales 20,000 By balance c/d

9,000

Cr. 20,000 4,000 1,000 25,000 Cr. 20,000

20,000 Sales Discount A/c Dr To Sales 1,000 By balance c/d

20,000 Cr. 1,000

1,000

1,000

120

Purchase Discount A/c Dr To balance c/d 200 By purchases 200 Cr.

200 Rent expenses A/c Dr To cash A/c 2,000 By balance c/d 2,000 Insurance expenses A/c Dr To cash A/c 3,000 By balance c/d 3,000 Advertisement expenses A/c Dr To cash A/c 5,000 By balance c/d 5,000

200

Cr.

2,000 2,000

Cr.

3,000 3,000 Cr.

5,000 5,000

121

Commission income A/c Dr To balance c/d 6,000 By cash A/c 6,000 Cr.

6,000 Rent income A/c Dr To balance c/d 4,000 4,000 By cash A/c

6,000

Cr. 4,000 4,000

122

M/s Dalvi Co. Ltd. Trial balance as at 31. 12. 2001 Particulars Cash 12 %preference share capital Share premium Preference dividend paid Equity share capital Disc. on issue of debentures 9 % debentures Interest on debentures paid Purchases Sundry creditor Purchase returns Rent expenses Insurance expenses Advertisement expenses Commission income Rent income 2,000 3,000 5,000 6,000 4,000 9,000 10,000 3,800 2,000 5,000 1,00,000 6,000 1,00,000 Debit (Rs.) 2,35,000 50,000 5,000 Credit (Rs.)

123

Sales Debtors Purchase discount Sales discount Total 1,000 2,96,000 20,000

25,000

200

2,96,000

124

M/s Dalvi Co. Ltd. Income statement for the year ending 31.12.01 Rs. Net Sales (per schedule 1) Less : Purchase cost of goods sold (per schedule 2) GROSS TRADING PROFIT Less : Non-trading expenses (per schedule 3) OPERATING PROFIT Add: Income other than sales (per schedule 4) Profit Before Interest and Income Tax Less : Finance Cost Profit Before Tax Less : Provision for income tax @ 30% PAT (available for appropriations) Less : Preference share dividends @ 12% Amount available for equity shareholders Less: Provision for equity dividends Less : Transfer to general reserves Less : Surplus (Profit and loss account credit balance) BALANCE 24,000 (6,600) 17,400 (10,400) 7,000 18,500 25,500 (14,000) 11,500 (3,450) 8,050 (6,000) 2,050 (1,025) (512.50) (512.50) NIL

125

Schedule 1: Showing Net Sales Rs. Sales Less: Sales return Less: Sales discount Net Sales Schedule 2: Showing Trading cost of goods sold Rs. Opening Stock Purchases Less: Purchase returns Less: Purchase discount NET PURCHASES Purchase cost of goods available for sale Less: Closing Stock (cost price or market price whichever is lower) Purchase cost of goods sold Schedule 3: Showing Non-trading expenses Rs. Rent expenses Add: Outstanding rent Insurance expenses Less: Prepaid insurance 2,000 1,000 3,000 (1,400) 1,600 3,000 Rs. 10,000 (2,000) (200) 7,800 7,800 (1,200) 6,600 Rs. NIL 25,000 Nil (1,000) 24000

126

Bad debts Provision for doubtful debts @ 10% of (Rs. 20,000 Rs. 2,000) Advertising Expenditure Less: Amount to be deferred (3/5th)

2,000 1,800 5,000 (3,000) 2,000 10,400

Schedule 4: Showing Income other than Sales Commission income Add: Accrued commission income Rent income Less: Pre-received rent income Rs. 6,000 1,000 4,000 (1,500) Rs. 16,000 2500 18,500

Schedule 5: Showing Finance Cost Interest on 9% debentures Discount on issue of debentures Schedule 6: Showing Shareholders Funds Rs. Equity share capital 12 % preference share capital Rs. 1,00,000 50,000 1,50,000 Rs. 9,000 5,000 14,000

127

Reserves and Surplus General Reserves Surplus (Profit and loss account credit balance) Share premium 512.5 512.5 5,000 6,025 6,025 Schedule 7: Showing Loan funds Secured debentures Rs. 1,00,000 1,00,000

Schedule 8: Showing Fixed Assets Fixed assets Schedule 9: Showing Investments Investments Schedule 10: Showing Current Assets Rs. Cash Debtors 20,000 Rs. 2,35,000 Rs. Nil Nil Rs. Nil Nil

128

Less: Bad debts Less: Provision for doubtful debts Stock (cost price or market price whichever is lower) Accrued commission income

(2,000) (1,800) 16,200 1,200 10,000 2,62,400

Schedule 11: Showing Loans and Advances Prepaid insurance Rs. 1,400 1,400

Schedule 12: Showing Miscellaneous Assets (to the extent not written off) Advertising expenditure (3/5 of 5000) Rs. 3,000 3,000

Schedule 13 : Showing Current liabilities and Provisions Rs. Current Liabilities Sundry Creditors Outstanding rent expense Pre-received income Provisions Provision for income tax Provision for dividends on equity shares 3,800 1,000 1,500 3,450 1,025 Rs.

6,300 4,475 10,775

129

Horizontal Format (per Sec. 211, Schedule VI, Companies Act) Balance Sheet as on 31.12.01 Liabilities Owners Funds (as per schedule 6) 1,56,025 Rs. Assets Net Fixed Assets (per schedule 8) Rs. Nil

Loans (as per schedule 7)

1,00,000 Investments (per schedule 9)

Nil

Current Liabilities and Provisions (as per schedule 13) 10,775

Current Assets (per schedule 10)

2,62,400

Loans and Advances (per schedule 11) Miscellaneous Assets (per schedule 12) 2,66,800

1,400 3,000 2,66,800

130

Vertical Format (per Section 211, Schedule VI, Companies Act) M/s Dalvi Co. Ltd. Balance Sheet as on 31.12.01 Schedule No. I. Sources of Funds 1) Shareholders funds (a) Capital (b) Reserves and Surplus 2) Loan Funds TOTAL II. Application of Funds 1. Net Fixed Assets 2. Investments 3. Currents Assets, Loans and Advances a. Current Assets b. Loans and Advances Less : Current Liabilities and Provisions Net Current Assets 4. Miscellaneous Current Assets (to the extent not written off or adjusted) TOTAL 12 10 11 13 8 9 6 6 7

Rs.

Rs.

1,50,000 6,025 1,56,025 1,00,000 2,56,025 Nil Nil 2,62,400 1,400 2,63,800 (10,775) 2,53,025 3,000 2,56,025

131

Assignment 6 M/s Ashutosh M/s Ashutosh Co. Ltd., enters into the following transactions for the period ending 31.12.2002 1. Issued 10,000 equity shares of Rs10 at Rs.12 2. Issued 1,000 10% Preference Shares of Rs. 100 at Rs. 90 3. Procured 15% loan from SBI for Rs. 50,000 4. Purchased goods on credit worth Rs. 25,000 5. Purchased goods on cash worth Rs. 20,000. 6. Sold goods on credit worth Rs. 30,000. 7. Sold goods on cash worth Rs. 20,000 after allowing a discount of 20%.

Required:

Pass Journal entries and post them in the ledger accounts. Prepare a Trial Balance as on 31.12.02.

132

HINT : Transaction No 1 (which is new) is to be understood as follows: Assets = Liabilities Cash Inflow = Rs. 1,20,000 The Journal entry for the above transaction will be as follows: Cash A/c Dr Rs.1,20,000 To Equity Share Capital a/c To Securities Premium a/c Rs. 1,00,000 Rs. 20,000 + Capital Equity Share Capital Rs. 1,00,000 Securities Premium = Rs. 20,000

Transaction No. 2 (which is also new) is to be understood as follows: Assets Cash Inflow = Rs. 90,000 Discount on Issue of Preference Shares = 10,000 The Journal entry will be as follows: Cash A/c Dr. Discount on issue of preference shares a/c Dr To Preference Share Capital a/c = Liabilities + Capital Preference Share Capital Rs. 1,00,000

Rs. 90,000 Rs. 10,000 1,00,000

133

Part 2 Required: Prepare an Income Statement and Balance Sheet for the year 2002.

Solution to Assignment no 6 M/s Ashutosh Co. Ltd. Journal Entries


Date Particulars Cash A/c To equity share capital A/c To share premium A/c Cash A/c Discount on the issue of preference shares A/c To 10% preference share capital A/c Cash A/c To 15% SBI loan A/c Purchase A/c To creditors A/c Purchase A/c To cash A/c Debtors A/c To sales A/c Cash/ A/c Discount allowed A/c To sales A/c Dr L/F Debit (Rs.) 1,20,000 1,00,000 20,000 Dr Dr 90,000 10,000 1,00,000 Dr Dr Dr Dr Dr 50,000 50,000 25,000 25,000 20,000 20,000 30,000 30,000 16,000 4,000 20,000 Credit (Rs.)

134

Ledger accounts All figures in rupees Cash A/c Dr To equity share capital To share premium To 10% PSC To 15% SBI loan To sales 1,00,000 20,000 90,000 50,000 16,000 By balance c/d 2,76,000 Equity share capital A/c Dr To balance c/d 1,00,000 By Cash/ Bank 1,00,000 Cr 2,56,000 2,76,000 By purchases 20,000 Cr

1,00,000 10% Preference share capital A/c Dr To balance c/d 1,00,000 By Cash/ Bank

1,00,000

Cr 1,00,000

1,00,000

1,00,000

135

15% SBI A/c Dr To balance c/d 50,000 50,000 By Cash/ Bank 50,000 50,000 Cr

Share Premium A/c Dr To balance c/f 20,000 By cash/ bank 20,000 Cr

20,000

20,000

Discount on issue of preference shares A/c Dr To 10% P.S.C 10,000 10,000 Purchase A/c Dr To Creditors To cash / bank A/c 25,000 20,000 45,000 45,000 By balance c/f 45,000 Cr By balance c/d 10,000 10,000 Cr

Creditors A/c Dr Cr

136

To balance c/f

25,000 25,000 Debtors A/c

By purchases

25,000 25,000

Dr To sales 30,000 By balance c/d 30,000 Sales A/c Dr To balance c/d 50,000 By debtors By cash sales By disc allowed 50,000 30,000 16,000 4,000 50,000 30,000 30,000

Cr

Cr

Dr To sales

Discount allowed A/c 4,000 By balance c/d 4,000 4,000 4,000

Cr

137

M/s Ashutosh Co. Ltd. Trial balance as at 31.12.2001 Particulars Cash/ bank Equity share capital 10% Preference share capital 15% SBI loan Share premium Discount on issue of preference shares Purchases Creditors Debtors Sales Discount allowed Total M/s Ashutosh Company Ltd. Income Statement for the Period Ending 31.12.2002 Gross Sales Less: - Sales returns Less: - Sales discount 4,000 3,45,000 3,45,000 30,000 50,000 10,000 45,000 25,000 Debit (Rs.) 2,56,000 1,00,000 1,00,000 50,000 20,000 Credit (Rs.)

Rs. 50,000 Nil (4,000)

138

Net Sales Less: - Trading cost of goods sold ( as per Schd. 1 ) Gross Trading Profit Less: - Non-trading expenses ( As per Schd. 2 ) Operating Loss Add: - Income other than sale Accrued rent income Profit Before Interest and Tax (PBIT) Less: - Finance costs (as per Schd. 3) Profit Before Tax (PBT) Less: - Provision for income tax (30 % of 28,850) Profit After Tax (PAT) (Available For Appropriation *) Less: - Provision for preference share dividend @ 10 % Profit available for Appropriation Less: - Provision for equity share dividend @ 20 % Balance Transfer to General Reserve M/s Ashutosh company Ltd. Balance Sheet as on 31st Dec. 2002 Application of Funds Fixed assets Net current assets Rs.

46,000 (44,200) 1,800 (5,450) (3,650) 50,000 46,350 (17,500) 28,850 (8,655) 20,195 (10,000) 10,195 (2,039) 8,156 Rs.

Nil

139

Current assets (As per Schd. 4) Less: - Current liabilities (as per Schd. 5) Net Assets (Total) Sources Of Funds Owners funds Share capital Equity share capital Preference share capital Reserves General reserve Share premium Borrowed funds SBI loan Total Schedule 1 : Trading Cost of Goods Sold Opening Stock Add: - Purchases Gross Purchases Less: - Purchase returns Less: - Purchase discount Purchase cost of goods available for sale Less: - Closing stock (cost or market price, whichever is less)

3,33,350 (55,194) 2,78,156 2,78,156 Rs. Rs.

1,00,000 1,00,000 8,156 20,000 28,156 50,000 2,78,156 Rs. Rs. Nil 45,000 Nil Nil 2,00,000

45,000 45,000 (800)

140

Trading Cost of Goods Sold

44,200

Schedule 2 : Non-Trading Expenses Outstanding expenses Bad debts Provision for doubtful debts [10 % of (30,000 - 500)] Non-trading expenses

Rs.

Rs. 2,000 500 2,950 5,450

Schedule 3 : Finance costs Interest on SBI loan (15 % of 50,000) Discount on issue of preference shares (written-off) Finance costs

Rs.

Rs. 7,500 10,000 17,500

Schedule 4: Current Assets Cash Closing stock (cost or market price, whichever is less) Sundry debtors Less: - Bad debts Less: - Provision for doubtful debts Accrued rent income Current assets

Rs.

Rs. 2,56,000 800

30,000 (500) (2,950)

26,550 50,000 3,33,350

141

Schedule 5 : Current Liabilities Outstanding expenses Sundry creditors Outstanding interest Provision for income tax Provision for preference dividend Provision for equity dividend Current liabilities

Rs.

Rs. 2,000 25,000 7,500 8,655 10,000 2,039 55,194

142

Assignment 7- Abacus Airways Ltd. Part 1 The following transactions have been entered into by Abacus Airways Ltd., for the year ending 31.03.1998. Sr. No. 1 2 3 4 Date 1.1.1997 2.2.1997 3.3.1997 10.4.1997 Particulars Brought in capital Procured loan from British Bank of Middle East (BBME) Purchase of aircraft (on credit) Purchase of fixed assets: Buildings 5 15.4.1997 Simulator Purchased stocks (on credit): In-flight consumables Engineering spares 6 20.4.1997 Stationery Cash received on sales of tickets: Pax 7 8 9 25.4.1997 25.4.1997 30.5.1997 Cargo Agents sold tickets to Pax for cash Commission to agent on account of transaction 7 is 9% Agents sold cargo space for cash, for shipments, to send goods from LON- DXB 10 11 20.6.1997 30.9.1997 Commission to agent from above Commission income received for pilot training and consultancy Direct Operating Costs (DOCs) Aviation fuel Catering 1 50 60 40 In Rupees (million) 1,000 1,000 800 80 300 80 100 20 300 50 100 10

143

Landing and parking 12 25.10.1997 Other DOCs Aircraft related expenses: Insurance 13 30.11.1997 Bank guarantee charges Overheads: Employee related Postage and communications 14 15 16 11.2.1997 25.3.1998 25.3.1998 Promotions Interest on loans from BBME @ 6% per annum paid for 8 months Flown revenue (pax) 60% of cash and credit sales Flown Revenue (cargo) 55 %

30 20 30 6 10 2 8

From the following particulars prepare the following: Journal Ledger Trial balance

144

Part 2 It is now 31.03.98. You are now told that: Outstanding DOCs on account of landing and parking 10 Mn rupees Accrued Income on account of pilot training 25 Mn rupees Prepaid Insurance 10 Mn rupees 15% of the total sales seems to be irrecoverable. Outstanding interest on BBME loan has to be accounted for. Depreciation Straight Line Method:
Aircraft

12 years 15 years 4 years

Building

Simulator

Market price of stocks has gone down by 20% Provision for income tax @ 5% Provision for dividends @ 20%.

Required: Prepare financial statements of the year 1998.

145

Solution to Assignment no. 7 In the books of Abacus Airways Journal entries


In Rupees (million)
Sr. No. 1 Date 1. 1. 97 Code Transaction details Cash A/c Dr LF Debit 1,000 1,000 1,000 1,000 800 800 80 300 380 200 200 350 300 50 91 Dr 9 100 9 Credit

To Capital A/c 2 2. 2 .97 Cash A/c Dr

To BBME loan A/c 3 3 .3 .97 Aircraft A/c Dr To Accounts Payable A/c 4 10 .4. 97 Buildings A/c Dr Simulators A/c Dr To Accounts Payable A/c 5 15. 4. 97 Stocks A/c Dr To Accounts Payable A/c 6 20 .4. 97 Cash A/c Dr To Forward Pax sales To Forward Cargo sales 7 25 .4 .97 Agents Pax A/R A/c Dr Forward Pax commission A/c 8 30 .5 .97 To Forward Pax Sales Agents Cargo A/R A/c Dr

146

Forward Cargo comm. A/c Dr 9 10 20 .6. 97 30 .9 .97 To Forward Cargo Sales A/c Cash A/c Dr To Commission Income (Pilot T&C) Aviation fuel A/c Dr Catering A/c Dr

1 10 50 50 60 40 30 20 150 30 6 36 10 2 8 20 40 40 240 240 5.4 5.4 33 33 0.55 0.55

Landing & Parking A/c Dr Other DOCs A/c Dr 11 25.10. 97 To Cash Insurance exp. A/c Dr Bank guar. Chgs. A/c Dr 12 30. 11. 97 To Cash Employee related cost A/c Dr Post & Comm. Exp. A/c Dr Promotions A/c Dr To Cash 13 14 1 .12 .97 25 .3 .98 Interest expense A/c Dr To Cash Forward pax sales A/c Dr To Flown pax revenue A/c Flown pax comm. A/c Dr 15 25 .3. 98 To Forward pax commn. A/c Forward Cargo Sales A/c Dr To Flown cargo revenue A/c Flown cargo commn. A/c Dr To Forward cargo commn.

147

In the books of Abacus Airways Ledger Accounts In Rupees (million) Dr Date 1.1. 97 Particulars To capital Cash Amount 1,000 Date 30 .9. 97 Particulars By aviation fuel 60 By catering 40 By L & P 30 By other DOCs - 20 2. 2. 97 To BBME loan 1,000 25. 10. 97 By insurance exp. 30 By bank chgs. - 6 20 .4 .97 To forward pax sales 300 To forward cargo sales - 50 20. 6 .97 To commission income 50 25 .3 .98 2,400 350 By employee R. C. 10 By post & Com. By promotion By interest exp. By bal. c/d 40 2,154 2,400 20 36 Amount 150 Cr

148

Dr Date 25. 3. 98 Particulars To balance

Accounts Payable Account Amount 1,380 Date 30 .9 .97 Particulars By aircraft By building 80 By simulator - 300 Amount 800 380

Cr

Date

Particulars

Amount

Date

Particulars By stocks

Amount 200 1,380

1,380

Dr Date 25. 3. 98 Particulars To balance

Capital Amount 1,000 1,000 Date 1. 1. 97 Particulars By cash Amount 1,000 1,000

Cr

Dr Date 25. 3 .98 Particulars To balance

BBME Loan Amount 1,000 1,000 Date 2. 2. 97 Particulars By cash Amount 1,000 1,000

Cr

149

Dr Date 3 .3 .97 Particulars To A/P

Aircraft Amount 800 800 Date 25. 3. 98 Particulars By balance Amount 800 800

Cr

Dr Date 10. 4 .97 Particulars To A/P

Buildings Amount 80 80 Date 25 .3.98 Particulars By balance Amount 80 80

Cr

Dr Date 10 .4 .97 Particulars To A/P

Simulator Amount 300 300 Date 25. 3 .98 Particulars By balance Amount 300 300

Cr

Dr Date 15 .4 .7 Particulars To A/P

Stocks Amount 200 200 Date 25. 3. 98 Particulars By balance Amount 200 200

Cr

150

Dr Date 25. 3. 98 25. 3 .98 Particulars To flown pax revenue To balance

Forward Pax Sales Amount 240 160 Date 20.4. 97 25. 4 .97 Particulars By cash By agents By commission 400 Amount 300 91 9 400

Cr

Dr Date 25.3. 98 25. 3. 98 Particulars To flown cargo revenue To balance

Forward Cargo Sales Amount 33 27 Date 20. 4. 97 30. 5. 97 Particulars By cash By agents By commission 60 Amount 50 9 1 60

Cr

Dr Date 20 .4 .97 Particulars To forward Pax sales

Agents Pax A/R Amount 91 91 Date 25. 3 .98 Particulars By balance Amount 91 91

Cr

151

Dr Date 25. 4 .97 Particulars To forward Pax sales

Forward Pax Commission Amount 9 Date 25 .3. 98 25. 3. 98 9 Particulars By flown Pax commission By balance Amount 5.4 3.6 9

Cr

Dr Date 30. 5. 97 Particulars To forward cargo sales

Agents Cargo A/R Amount 9 9 Date 25 .3 .98 Particulars Amount B9 9

Cr

Dr Date 30. 5. 97 Particulars

Forward Cargo Commission Amount Date 25. 3. 98 Particulars By flown cargo commission By balance 1 Amount 0.55 0.45 1

Cr

To forward cargo 1 sales

152

Dr Date 30. 5 .97 Particulars To balance

Commission Income Pilot T & C Amount 50 50 Date 20. 6. 97 Particulars By cash Amount 50 50

Cr

Dr Date 30 .9. 97 Particulars To cash

Aviation Fuel Amount 60 60 Date 25. 3. 98 Particulars By balance Amount 60 60

Cr

Dr Date 30. 9. 97 Particulars To cash

Catering Amount 40 40 Date 25 .3 .98 Particulars By balance Amount 40 40

Cr

Dr Date 30. 9. 97 Particulars To cash

Landing & Parking Amount 30 30 Date 25. 3. 98 Particulars By balance Amount 30 30

Cr

153

Dr Date 30 .9 .97 Particulars To cash

Other DOCs Amount 20 20 Date 25. 3. 98 Particulars By balance

Cr Amount 20 20

Dr Date 25. 10. 97 Particulars To cash

Insurance Expenses Amount 30 30 Date 25. 3 .98 Particulars By balance

Cr Amount 30 30

Dr Date 25 .10. 97 Particulars To cash

Bank Guarantee Charges Amount 6 6 Date 25. 3 .98 Particulars By balance

Cr Amount 6 6 Cr Date 25. 3. 98 Particulars By balance Amount 10 10

Dr Date 30 .11. 97 Particulars To cash

Employee Related Cost Amount 10 10

154

Dr Date 30. 11. 97 Particulars To cash

Postage & Communications Amount 2 2 Date 25. 3 .98 Particulars By balance

Cr Amount 2 2

Dr Date 30. 11. 97 Particulars To cash

Promotions Amount 8 8 Date

Cr Particulars By balance Amount 8 8 Cr Date 25. 3 .98 Particulars By balance Amount 40 40

25 .3. 98

Dr Date 30. 11 .97 Particulars To cash

Interest Expenses Amount 40 40

Dr Date 30. 11. 97 Particulars To balance

Flown Pax Revenue Amount 240 240 Date

Cr Particulars By forward pax sales Amount 240 240

25. 3 .98

155

Dr Date 30 .11. 97 Particulars To forward Tax commission

Flown Pax Commission Amount 5.4 5.4 Date 25. 3. 98 Particulars By balance

Cr Amount 5.4 5.4 Cr

Dr Date 30 .11.97 Particulars To balance

Flown Cargo Revenue Amount 33 33 Date 25. 3. 98 Particulars

Amount 33 33

By forward cargo sales

Dr Date 30. 11. 97 Particulars

Flown Cargo Commission Amount 0.55 0.55 Date 25. 3 .98

Cr Particulars By balance Amount 0.55 0.55

To forward cargo commission

156

M/s Abacus Airways Trial balance as on 31. 3. 1998 Sr. No Name of Account Cash A/P Capital BBME Loan Aircraft Buildings Simulator Stocks Forward Pax. Sales Forward Cargo Sales Agents Pax. A/R Forward Pax. Commission Agents Cargo A/R Forward Cargo Commission Commission Income Pilot T & C Aviation Fuel Catering Landing & Parking Other DOCs Insurance Expense Bank Guarantee Charges 60 40 30 20 30 6 91 3.6 9 0.45 50 800 80 300 200 160 27 Debit balance 2,154 1,380 1,000 1,000 In Rupees Mn. Credit balance

157

Employee Related Cost Postage & Communications Promotions Interest Expense Flown Pax. Revenue Flown Pax. Commission Flown Cargo Revenue Flown Cargo Commission Total balance Abacus Airways

10 2 8 40 240 5.4 33 0.55 3,890 3,890

Income Statement for the period ending 31.03.1998 Particulars Revenue Flown Passenger revenue Flown cargo revenue Commission Income Accrued Income Expenses Aviation fuel Catering Landing and parking (30+10) Other DOCs 60 40 40 20 240 33 50 25 348 Amount (in Rs.) Amount (in Rs.)

158

Insurance expense (30-10) Bank charges Employee related cost Postage & communication Promotion Interest expense (40+20) Flown passenger communication Flown cargo communication Bad debt expense Depreciation expense Depreciation expense on stock Profit before tax Less: Provision for tax @ 30% Profit after tax Less: Provision for dividend @ 20% Net income (loss)

20 6 10 2 8 60 5.4 0.55 16.5 147 40 475.45 (127.45) Nil (127.45) Nil (127.45)

Balance sheet as at 31. 03. 1998


Assets Current assets Amount in mn. Rs. Amount in mn. Rs.

159

Cash Agents A/R (100+25-16.5) Stocks (200-40) Prepaid insurance Forward commission (3.6+0.45) Fixed assets Aircraft (800-66.67) Buildings (80-5.33) Simulator (300-75) Total assets Liabilities and capital Current liabilities A/P Outstanding DOC Outstanding interest Provision for Income tax Provision for dividend Forward passenger sales Forward cargo sales Long term liabilities British bank loan Capital Reserves Total liabilities

2,154 108.5 160 10 4.05 733.33 74.67 225 1,033 3,469.55 2,436.55

1,380 10 20 Nil Nil 160 27 1,597 1,000 1,000 (127.45) 3,469.55

160

Assignment 8 M/s PQR


Part 1 M/s PQR., have entered into the following transactions for the period ending 31.3.2001. Sr. Particulars no. 1. Owners bring in equity share capital 2. Borrowed loan (long term) on 1st April 2000 at 10% Interest (interest payable half-yearly on 1st Oct. 2000 and 1st April 2001 respectively) 3. Invested in preference shares of a company on 1st April 2000 and at 10% dividend 4. Purchased fixed assets for cash 5. Purchased fixed assets (on credit) 6. Purchased stock for cash from Mr. A 10000 units @ Rs.10/- per unit from Mr. B 6000 units @ Rs.15/- per unit 7. Purchased on credit from Mr. D, Mr. E and Mr. F from Mr. D 6000 units @ Rs. 20/- per unit from Mr. E 5000 units @ Rs.25/- per unit from Mr. F 11000 units @ Rs.30/- per unit 8. Purchased returns to Mr. D 1000 units to Mr. F 1500 units 9. Paid creditors: Mr. D Rs.48,000 in full satisfaction of Rs.50,000 (actual amount) Mr. E Rs.85,000 in full satisfaction of Rs.90,000 Mr. F Rs. 1,30,000 in full satisfaction of Rs. 1,50,000 10. Sales for cash Mr. M: 5000 units @ Rs. 100/- per unit Mr. N: 6000 units @ Rs. 120/- per unit. Allowed a cash Discount of 10% Sales on credit to X, Y and Z To Mr. X: 8000 units @ Rs.200/- per unit Mr. Y: 5000 units@ Rs.250/- per unit Rs. Lakhs Rs. 50 Rs.30 Rs. 1 Rs.10 Rs. 5

161

11. 12.

13. 14. 15. 16. 17. 18. 19.

Mr. Z: 10000 units @ Rs.300/- per unit Sales returns from cash sales party Mr. M 200 units Sales returns from Mr. X: 300 units Mr. Z: 400 units Received cash from customers: (actual amounts). Mr. X: Rs. 9,80,000 in full satisfaction of Rs. 10,00,000 Mr. Y: Rs. 7,50,000 in full satisfaction of Rs. 8,00,000 Mr. Z: Rs. 20,00,000 in full satisfaction of Rs. 22,00,000 Rent expense incurred Insurance expense paid Commission income received Rent income received Advertising expense incurred Preliminary expenses Bad debts

Rs. 5 Rs. 7 Rs.10 Rs.20 Rs. 6 Rs. 3 Rs. 1

Required: Part 1 A.Journalize the above transactions (for practice). B.Post them into the ledger (for practice). C.Prepare a trail balance as on 31.03.01.

162

Part 1 C The Trial Balance of PQR Ltd., as on 31.3.01 is presented below: M/S PQR LTD TRIAL BALANCE AS ON 31.03.01 Sr. no. 1 2 3 4 5 6 7 8 9 10 11 12 13 14 15 16 17 18 19 20 21 22 Name of Account Owners capital Long term creditors Investment (Pref. shares) Fixed assets Sundry creditors Purchases Purchases returns Sales Cash discount Cash Sales returns Sales discount Rent paid Debtors Purchase discount Preliminary expenses Bad debts Insurance Commission income Rent received Advertising expense Interest expense TOTAL Debit Rs. Lakhs Credit Rs. Lakhs 50.00 30.00 7.20 7.65 0.65 70.70 0.72 120.55 2.00 2.70 5.00 15.70 0.27 3.00 1.00 7.00 10.00 20.00 6.00 1.50 188.82 188.82

1.00 15.00

163

Part 2 It is now 31.03.01 You are given the following adjustments for the year-ended 31.03.01.
Market

price of closing stock is down by 20% in relation to cost price as on 31.03.01 is @ 25% of fixed assets rent expenses Rs.2

Depreciation Outstanding 60%

of the insurance paid expired in the current period bad debts Rs.0.5 income pre-received Rs. 3 lakhs for doubtful debts @ 10%

Additional Provision

Commission Accrued

rent income Rs. 4 lakhs for dividend 50% of the profit available for distribution expenditure to be changed over 2 years expenses to be amortised (depreciated) over 3 years

Provision

Advertising Preliminary Provision

for income tax @ 30%.

With the help of the above trial balance as on 31.03.01 and the above adjustments, you are required to a. Prepare Income and other statements. b. Prepare a Balance sheet as per schedule VI of the Companies Act. c. Spell out the significant accounting policies followed by you.

164

Solution to Assignment no. 8


M/s PQR Ltd. Income Statement for the year ending 31.03.01 Particulars A Sales Less: Sales returns Less: Sales discount NET SALES B Trading Cost of Sales (as per schedule 2) GROSS TRADING PROFIT D Other expenses (as per schedule 3) E Operating Profit F Income other than sales (as per schedule 4) G H I J K L M PROFIT BEFORE INTEREST AND TAXES Less: Finance costs as per schedule 5 PROFIT BEFORE TAX Less: Provision for income tax at 30% of 65.26 PROFIT AFTER TAX (Net Profit) Less: Provision for dividend at 50% BALANCE TRANSFERRED TO GENERAL RESERVE

Rs. Lakhs. 70.70 (2.00) (3.42)

Rs. Lakhs.

C (A B)

65.28 (6.15) 59.13 (20.97) 38.16 31.10 69.26 (4.00) 65.26 (19.58) 45.68 (22.84) 22.84

165

Schedule 1: Valuation of Closing Stock Particulars Purchases A No. of units 10,000 6,000 6,000 5,000 11,000 38,000 ===== 1,000 1,500 2,500 ===== 35,500 5,000 6,000 8,000 5,000 10,000 34,000 ===== 2,00 3,00 4,00 900 ===== 33,100 ===== Cost per unit Rs. 10 15 20 25 30 Total Amt Rs. Lakhs 1.00 0.90 1.20 1.25 3.30 7.65

Less Purchase Returns

20 30

0.20 0.45 0.65 7.00

Net Purchases C=A-B Sales D

Less Sales returns E

Net sales F= D-E

166

Closing stock (C-F) (Net purchases less Net sales) Closing stock Valued using FIFO Method Closing stock NB: 2400 units @ Rs. 30 per unit Less: 20% depreciation in stock Round off and valued at Rs. 0.58 lakhs

2,400 24 2,400 0.58 0.58

= Rs 7,200 1,440 5,840

Schedule2: Showing Trading Cost of Goods Sold Rs. Lakhs Net units purchased (as per schedule 1) 35,500 Less Purchase discount Less Closing stock (as per schedule 1) Cost of goods sold Cost per unit 7.00 0.27 0.58 6.154

167

Schedule 3: Showing Total Expenses Particulars Rent Add: Outstanding rent Insurance Less: Prepaid insurance Advertising expenses Less: Deferred Portion Bad debts Add: Additional bad debts Provision for doubtful debts Debtors 15.7 Less Bad debts 0.5 15.2 Provision for doubtful debts @ 10% 1.52 Depreciation at 25% of fixed assets (Rs.15 lakhs * 0.25) Total Expenses Cost of Goods Sold = Opening Stock + Purchases Closing Stock 1.52 3.75 20.97 Amt. Rs. Lakhs 5 +2.00 ========= 7.00 (2.80) ========= 6.00 (3.00) ========= 1.00 +0.5 ========= 7.00 Amt. Rs. Lakhs

4.20 3.00

1.50

168

Schedule 4: Showing Income other than Sales Commission income Less: Pre-received commission Rent Income Add: Accrued Rent Dividends on Preference shares investment

10 (3) 20 4 24 0.1 31.1 7.0

Schedules 5: Showing Finance Costs Interest Expenses paid (1.10.2000) Outstanding (1.4.2001) Preliminary Expenses Less: Deferred portion (to the extent not written off)

1.5 1.5 3.0 (2.0) 1.0 4.0 3.0

BALANCE SHEET M/S PQR LTD., AS ON 31.03.01 Liabilities Rs. Lakhs Rs. Lakhs FIXED ASSETS SHARE CAPITAL Owners capital 50.00 less : depreciation Assets Rs. Lakhs 15 (3.75) 11.25 Rs. Lakhs

169

RESERVES AND SURPLUS Profit c/f from P&L a/c SECURED LOANS AND ADVANCES Secured loans- Long term Unsecured loans

INVESTMENTS 22.49 Investment in preference shares CURRENT ASSETS, LOANS AND ADVANCES 30.00 Cash Closing stock as per schedule 1 Debtors 15.70 (0.50) 15.20 7.20 2.00 1.50 19.28 22.49 PREPAID INSURANCE ADVERTISING EXPENDITURE 52.47 DEFERRED PRELIMINARY EXPENSES ACCRUED DIVIDEND 154.96 2.00 0.10 154.96 2.80 3.00 Less: Provision for d. debts (1.52) 13.68 120.55 0.58 1.00

CURRENT LIABILITIES AND PROVISIONS Sundry creditors Outstanding rent Outstanding interest Provision for tax Provision for dividend

Less: bad debts

170

Вам также может понравиться